Vascular Flashcards

1
Q

Following a renal transplant, a 38-year-old woman on cyclosporine and prednisone develops
hypertension, encephalopathy, and seizures. Which of the following is the most likely cause of her
clinical syndrome?

A. Drug toxicity
B. Posterior reversible encephalopathy syndrome
C. Viral encephalitis
D. Subarachnoid haemorrhage
E. Reversible cerebral vasoconstrictor syndrome

A

Posterior reversible encephalopathy syndrome

Posterior reversible encephalopathy syndrome (PRES) is an acute condition which presents with
headache, confusion and seizures. Whilst the same is true for Subarachnoid hemorrhage,
Reversible Cerebral Vasoconstriction Syndrome (RCVS) and viral encephalitis, in this case the
history of renal transplant and immunosuppressants make PRES the more likely diagnosis. The exact pathophysiology of PRES is not known but includes a loss of cerebral autoregulation and hyperperfusion with break down of the blood brain barrier and cerebral odema. Imaging will demonstrate parietal and occpital lobe vasogenic oedema. Treatment is supportive, correcting the underying cause with many patients requiring admission to intensive care.

How well did you know this?
1
Not at all
2
3
4
5
Perfectly
2
Q

Sacral sparing of pain and temperature sensation is seen most often with which of the following
spinal cord syndromes?

A. Brown-Séquard syndrome
B. cauda equina syndrome
C. Central cord syndrome
D. anterior cord syndrome
E. posterior cord syndrome

A

Central cord syndrome

Sacral sparing of pain and temperature sensation is most often seen with central cord syndrome,
which typically effects upper extremities more than lower extremities. Brown-Séquard syndrome results in contralateral loss of pain and temperature sensation below the level of the lesion.
Anterior cord syndrome causes loss of pain and temperature at and below the level of injury.
Posterior cord syndrome is characterized by isolated loss of proprioception and vibratory
sensation. Cauda equina syndrome presents with perianal and saddle paresthesia with bowel,
bladder and/or sexual dysfunction.

How well did you know this?
1
Not at all
2
3
4
5
Perfectly
3
Q

Intravenous tissue plasminogen activator should be administered within a maximum of how many hours after a patient sustains an acute ischemic stroke?

A. 1.5
B. 3
C. 4.5
D. 6
E. 24

A

4.5

Both the American Stroke Association and European Stroke Association guidelines recommend administering IV alteplase within 4.5 hours of symptom onset. This includes wake-up strokes where the patient was last seen well <4.5hours ago. The 2014 meta-analysis by Emberson (cited in the ESO guidelines) contains 9 RCTs with 6700 patients and demonstrates a improved rate of mRS 0-1 outcomes at 3 months for all age groups and stroke severity up to 4.5 hours after symptom onset, however the groups receiving IV alteplase >4.5hr after symptom onset did not show a statistically significant benefit. A meta-analysis by Campbell of the EXTEND, EPITHET and ECASS4 trials did not show a benefit of alteplase after 4.5 hours when the wake up strokes were removed from analysis

How well did you know this?
1
Not at all
2
3
4
5
Perfectly
4
Q

Injury to the fornix can cause

A. poikilothermia
B. hemiplegia
C. anterograde amnesia
D. alexia
E. apraxia

A

anterograde amnesia

Isolated damage to the fornix has been reported to cause anterograde amnesia. Poikilothermia is
loss of normal thermoregulation that can result from hypothalamic injury. Hemiplegia is complete paralaysis of half of the body. It can be caused by a wide variety of medical conditions including brain and spinal cord injury. Alexia is a disorder of reading that is seen with injury of the dominant angular gyrus. Apraxia is a disorder of motor planning to perform tasks or movements. It is seen with posterior paretal cortex and corpus callosum injuries

How well did you know this?
1
Not at all
2
3
4
5
Perfectly
5
Q

Which of the following Fisher grades has the highest predictability of vasospasm when evaluating patients with a subarachnoid hemorrhage?
Answers:
A. Grade 0
B. Grade 1
C. Grade 2
D. Grade 4
E. Grade 3

A

Grade 3

Thick cisternal clot increases the risk of vasospasm. In the original Fisher scale, grade 3, thick cisternal clot, has the highest risk of vasospasm but in the modified scale, grade 4, thick cisternal clot with Intraventricular Hemorrhage (IVH), has the highest risk of vasospasm. In the original Fisher scale, no SAH nor IVH is grade 1 while diffuse thin SAH is grade 2 without clots, and grade 3 is locaized clots iand or layers of blood greater than one mm in thickness. IVH and ICH is grade 4. In the modified Fisher scale, focal or diffuse thin subarachnoid hemorrhage (SAH) with no IVH is grade 1, while focal or diffuse thin SAH with IVH is grade 2. Focal or diffuse thick SAH without IVH is grade 3 and focal or diffuse thick SAH with IVH is grade 4. No SAH seen on CT head is grade 0.

How well did you know this?
1
Not at all
2
3
4
5
Perfectly
6
Q

Which of the following types of aphasia is characterized by severely affected ability to repeat
words, relatively normal comprehension, awareness of the speech deficit, and fluent paraphasic
speech?
Answers:
A. anomic aphasia
B. expressive aphasia
C. transcortical sensory aphasia
D. global aphasia
E. conduction aphasia

A

conduction aphasia

Conduction aphasia is characterized by pronounced difficulty with repetion with intact comprehension and fluency in spontaneous speech. The lesion is located at the arcuate fasciculus. Patients with anomic aphasia have mild expressive language deficits in word finding especially specific nouns and verbs. Expressive (Broca’s) aphasia may result from injury to the dominant inferior frontal gyrus. Individuals with expressive aphasia have trouble speaking fluently but comprehension is relatively preserved. Transcortical sensory aphasia is an uncommon form of aphasia that may occur when a lesion functionally isolates Wernicke’s areas from the rest of the brain. This results in intact repetition and articulation, but compromised speech comprehension. Global aphasia is a severe form of nonfluent aphasia, caused by damage to the dominant hemisphere. Both receptive and expressive language skills as well as auditory and visual comprehension are compromised.

How well did you know this?
1
Not at all
2
3
4
5
Perfectly
7
Q

Which of the following is the most likely cause of an intracranial large vessel occlusion in a patient
with cryptogenic stroke?
Answers:
A. mitral stenosis
B. paroxysmal atrial fibrillation
C. thrombophilia
D. Arterial dissection
E. Atherosclerosis

A

paroxysmal atrial fibrillation

Paroxysmal atrial fibrillation is one of the most common underlying causes of cryptogenic stroke and large vessel occlusion. Cryptogenic stroke has been reported as an independent marker of paroxysmal atrial fibrillation. Antiplatelet therapy is the mainstay of treatment. The Trial of Org 10172 in Acute Stroke Treatment defines Acute Stroke Treatment defines cryptogenic stroke (CS) as a cerebral infarct not attributed to a definite source of cardioembolism, large-vessel atherosclerosis, or small-vessel disease, despite (1) extensive cardiac, vascular, hematologic, and serological evaluation; (2) evidence of >1 competing cause, or (3) incomplete diagnostic evaluation. The Causative Classification System definition requires a diagnostic evaluation, including brain and cerebrovascular imaging along with a cardiac evaluation. Causative Classification System divides cryptogenic stroke into 2 categories: cryptogenic embolism and other cryptogenic. Cryptogenic embolism refers to a stroke in which there is angiographic evidence of abrupt cutoff consistent with a blood clot within otherwise angiographically normal-looking intracranial arteries, imaging evidence of complete recanalization of previously occluded artery, or the presence of multiple acute infarctions that have occurred closely related in time without detectable abnormality in the relevant vessels. Other cryptogenic is reserved for those not fulfilling the criteria of cryptogenic embolism.

How well did you know this?
1
Not at all
2
3
4
5
Perfectly
8
Q

A 48-year-old man is evaluated because of dizziness, vertigo, and hoarseness after having
chiropractic neck manipulation. Examination shows nystagmus, paralysis of the right palate,
decreased sensation to pinprick on the right side of the face and left hemibody, and right-sided
ataxia. This patient most likely has a lesion in which of the following locations?
Answers:
A. Posterior Inferior Cerebellar Artery
B. Vertebral artery
C. Anterior spinal artery
D. Superior Cerebellar Artery
E. Basilar artery

A

Vertebral artery

The patient has lateral medullary syndrome which is most commonly due to occlusion of the Posterior Inferior Cerebellar Artery artery, followed by the vertebral artery. However arterial pathology following chiropractic neck manipulation is most likely vertebral artery dissection which makes this the best answer to this question. Lateral medullary syndrome (Wallenberg syndrome) is comprised of vertigo with nystagmus (due to inferior vestibular nucleus and pathways), ipsilateral Horner’s (due to sympathetic fibers), contralateral pain/temperature loss (due to spinothalamic tract), and dysphonia/dysphagia/dysarthria (due to different nuclei and fibers of the IX and X nerves). Sensory loss is the most common feature (90%) which can be a combination of hemibody and/or trigeminal distributions. The other symptoms are all found in 70-75% of cases.

How well did you know this?
1
Not at all
2
3
4
5
Perfectly
9
Q

Which of the following is clopidogrel bisulfate’s mechanism of action in stroke prevention?
Answers:
A. GPIIb/IIIa inhibitor
B. COX inhibitor
C. P2Y12 ADP antagonist
D. ATP analogue
E. PAR-1 antagonist

A

P2Y12 ADP antagonist

All answers are methods of platelet inhibition. The incorrect answers are: GPIIb/IIIa inhibitors (abciximab), COX inhibitors (aspirin), ATP analogues (cangrelor), and PAR-1 antagonists (vorapaxar). Clopiogrel is an irreversible, competitive P2Y12 ADP receptor antagonist. Initial platelet activation occurs through interaction of the platelet with the exposed subendothelium. Upon platelet activation ADP is released from platelet dense granules which creates a cascade of subsequent activation and recruitment to the platelet plug.

How well did you know this?
1
Not at all
2
3
4
5
Perfectly
10
Q

In a patient who has sustained an aneurysmal subarachnoid hemorrhage, what is the peak time for
development of symptomatic vasospasm?
Answers:
A. 1-3 days
B. 4-6 days
C. 7-10 days
D. 11-14 days
E. 15-18 day

A

7-10 days

The risk of vasospasm peaks at day 7-10. The risk of vasospasm is low on 1-3, then increases, reaching its peak from day 7-10. After day 10 the risk of vasospasm starts decreasing again. It is rare after day 14.

How well did you know this?
1
Not at all
2
3
4
5
Perfectly
11
Q

The Mini Mental State Examination (MMSE) tests the functionality of the right hemisphere by
which of the following methods?
Answers:
A. serial 7’s
B. orientation questions
C. copying a symbol
D. recall of objects
E. repeating the given sentence

A

copying a symbol

Copying symbols is a measure of constructional ability which is a function of the non-dominant parietal lobe. Serial 7s, orientation questions, object recall, and phrase repetition are all tests of the dominant hemisphere.

How well did you know this?
1
Not at all
2
3
4
5
Perfectly
12
Q

A 12-year-old boy is evaluated because of sudden severe headache, neck stiffness, and vomiting.
A CT scan of the head shows diffuse subarachnoid hemorrhage. Which of the following is the most
likely cause of these findings?
Answers:
A. ICA aneurysm
B. Vertebrobasilar dissecting aneurysm
C. ACA aneurysm
D. AVM
E. MCA aneurysm

A

ICA aneurysm

The correct answer is ICA aneurysm. Although most of our data regarding pediatric aneurysms comes from smaller retrospective series as these are so rare, there is general consensus that the locations for pediatric aneurysms differ from those seen in adults. The most comm site for a ruptured aneurysm in an adult is ACOM, followed by PCOM. In children, the most common site is the intracranial ICA, and in particular the ICA bifurcation. The incidence of ICA terminus aneurysms is estimated to be 5x higher in pediatric than in adult ruptured aneurysms. There is a higher incidence of giant aneurysms in the pediatric population, but these still account for a minority (10-20%) of cases in the literature. MCA aneurysm is incorrect. In the pediatric population, MCA aneurysms are approximately as commonly reported as ACA aneurysms and less common than ICA and vertebrobasilar aneurysms. There is some evidence that MCA aneurysms may be more commonly seen in children <1 year of age. AVM is incorrect. The most common cause of non-traumatic ICH in children is AVM. However, the question asks about diffuse SAH and ruptured AVM is not typically associated with SAH. Rupture of AVM-associated (feeding vessel or pre-nidal) aneurysms may result in SAH or pure IVH, in the case of a distal anterior choroidal or posterior medial choroidal artery feeding aneurysm, and so careful examination of non-invasive and angiographic images in cases of ruptured pediatric AVM is critical to identify the cases in which bleeding from the nidus is not the immediate cause of intracranial hemorrhage, be it intra-parenchymal, subarachnoid or intraventricular. ACA aneurysm is incorrect. In adults, ACOM aneurysms account for the highest single location of ruptured aneurysms. Distal ACA aneurysms are less common, and less commonly associated with rupture due to smaller vessel size and lower flow. They are a common location for mycotic aneurysms, however. In the pediatric population, ACA aneurysms are approximately as commonly reported as MCA aneurysms and less common than ICA and vertebrobasilar aneurysms. Overall, Acom aneurysms are thought to be the third most common location for pediatric ruptured aneurysms, occurring in about 12% of cases. Vertebrobasilar dissecting aneurysm is incorrect. Posterior circulation aneurysms are more common in children than in adults but are still less common than supraclinoid ICA aneurysms. There is a higher incidence of fusiform aneurysms in the pediatric population, but dissection is less likely to be a cause in this case due to the absence of trauma in the history. Spontaneous dissection causing aneurysmal dilatation or pseudoaneurysm in the pediatric population is rare, unless there is a concurrent history of a connective tissue disorder such as Ehlers-Danlos syndrome Type 4 or Marfan syndrome.

How well did you know this?
1
Not at all
2
3
4
5
Perfectly
13
Q

An increased risk for rupture of an asymptomatic intracranial aneurysm is most commonly
associated with which of the following disorders?
Answers:
A. Ehler-Danlos Type 4
B. Marfan syndrome
C. Pseudoxanthoma elasticum
D. Autosomal dominant polycystic kidney disease
E. Autosomal recessive polycystic kidney disease

A

Autosomal dominant polycystic kidney disease

The correct answer is autosomal dominant polycystic kidney disease (ADPKD). Autosomal recessive polycystic kidney disease is not associated with aneurysm formation. The other three options are all connective tissue disorders with an increased risk of aneurysm formation however are less common than ADPKD (1:2000). Amongst patients with ADPKD, 10% will have unruptured incidental aneurysm compared to 3% of the general population. The rupture risk is also x4 compared to standard unruptured incidental aneurysm.

How well did you know this?
1
Not at all
2
3
4
5
Perfectly
14
Q

Which of the following types of aphasia is characterized by preserved repetition?
Answers:
A. expressive aphasia
B. global aphasia
C. conduction aphasia
D. transcortical motor aphasia
E. receptive aphasia

A

transcortical motor aphasia

Transcortical motor aphasia resembles expressive aphasia but patients are able to repeat. The
other aphasia syndromes listed demonstrate impaired repetition.

How well did you know this?
1
Not at all
2
3
4
5
Perfectly
15
Q

The most reliable method of differentiating CSF rhinorrhea from other forms of rhinorrhea is
Answers:
A. target test
B. glucose oxidized test
C. chlorine concentration
D. beta-2 transferrin
E. glucose concentration

A

beta-2 transferrin

Beta-2 transferrin is only found in CSF, perilymph, and vitreous humor and is thus a marker with extremely high sensitivity and specificity. The target sign also called a double ring sign or a Halo sign is tested for by applying the fluid sample to filter paper. When positive, the target sign suggests that CSF is mixed with blood or nasal discharge as CSF moves away on the filter paper and blood moves closer, such that two rings are visible. The CSF glucose level from nasal or ear secretions can be used to test for CSF leak. Glucose oxidase strips show positive results when the sample has a concentration of glucose over 20 mg/dL. Nasal discharge has a normal concentration of 10 mg/dL of glucose, thus, if the glucose test is negative the presence of CSF can be ruled out. However, there is a high false positive rate since hyperglycemia, epithelial inflammation, and bacterial infection may confound the result. Identification of glucose in combination with chlorine concentration of 100 mEq/L in nasal secretion has also been reported for the identification of CSF.

How well did you know this?
1
Not at all
2
3
4
5
Perfectly
16
Q

Which of the following racial groups has the highest prevalence of intracranial atherosclerotic
disease?
A. caucasian
B. native american
C. african american
D. asian
E. no difference between racial grouops

A

african american

Intracranial atherosclerosis is considered to be more frequent in African American when compared to other racial groups. In the United States, it disproportionately affects African American, Latin Americans, and Asian Americans, and it is associated with multiple modifiable risk factors including smoking, hypertension, hyperlipidemia, and diabetes.

How well did you know this?
1
Not at all
2
3
4
5
Perfectly
17
Q

A healthy right-hand dominant 17-year-old boy has a generalized tonic-clonic seizure. MR image
of the brain shows an arteriovenous malformation of the left posterior superior temporal gyrus that
is 2.5 cm in diameter and has venous drainage into the basal vein of Rosenthal. Which of the
following is the most likely Spetzler-Martin grade?
Answers:
A. Grade 1
B. Grade 2
C. Grade 3
D. Grade 4
E. Grade 5

A

Grade 3

This patient receives 1 point for eloquent cortex, 1 point for deep venous drainage, and considering that the size of the AVM is less than 3 cm, the patient receives another 1 point. All together this patient has grade 3 AVM on the Spetzler-Martin grading system. If the venous drainage is superficial only, patient receives 0 points and if the area involved is non-eloquent, then the patient receives 0 points. Patients receive 2 points for AVM sizes between 3-6cm, and those with AVM sizes more than 6 cm receive 3 points. Spetzler-Martin grading system predicts the risk associated with surgical resection of the AVM, and the higher the grade the more the surgical risks.

How well did you know this?
1
Not at all
2
3
4
5
Perfectly
18
Q

Which of the following is the most appropriate first-line treatment for symptomatic intracranial
atherosclerotic disease?
Answers:
A. observation with tight blood pressure control
B. anticoagulation
C. Surgical intervention
D. endovascular treatment
E. antiplatelet medication

A

antiplatelet medication

Stict control of atherosclerotic risk factors and medical management with antithrombotic therapy preferably with antiplatelet medications is the first-line treatment of patients with symptomatic intracranial atherosclerotic disease. Open revascularization and endovascular therapy with balloon
angioplasty and stenting should be considered in elegible patients with failure of medical therapy.

How well did you know this?
1
Not at all
2
3
4
5
Perfectly
19
Q

A 24-year-old woman experiences pain in the anterior aspect of the neck after vigorous exercise.
Examination shows ptosis, miosis, and contralateral paresis. Which of the following is the most
appropriate diagnostic study?
Answers:
A. Diagnostic cerebral angiogram
B. CT angiography
C. MRI angiography
D. MRI
E. Carotid ultrasound

A

CT angiography

This young patient with sudden onset ptosis and miosis likely has carotid artery dissection due to trauma to the artery with vigorous exercise. This would cause anterior neck pain. Hemiparesis could be related to thromboembolic phenomenon or decreased perfusion. In an emergent setting CT/ CTA is the best modality for initial evaluation. CT angiogram (CTA) can be quickly obtained with a head CT to look for hemorrhage and stroke, and it is less invasive than cerebral angiography. MRI/ MRA are less sensitive for carotid dissection and more time consuming with limited availability at many hospitals. Diagnostic cerebral angiogram is an invasive test and more time consuming than CT/CTA and is generally not needed for the diagnosis of carotid dissection. Carotid ultrasound is also a less sensitive modality than CTA for carotid dissection.

How well did you know this?
1
Not at all
2
3
4
5
Perfectly
20
Q

A 55-year-old man with paroxysmal atrial fibrillation is brought to the emergency department two
hours after the acute onset of left facial droop, hemiparesis, and slurred speech. The patient is on
aspirin therapy. CT scans of the head are shown. Which of the following is the most appropriate
initial step in management?
Answers:
A. intravenous thrombolysis alone
B. intraarterial thrombolysis alone
C. endovascular intervention alone
D. medical management
E. intravenous thrombolysis followed by endovascular intervention

A

intravenous thrombolysis followed by endovascular intervention

Intravenous thrombolysis is recommended in eligible patients presenting within 4.5 hours of symptom onset. Mechanical thrombectomy is recommended in patients with large vessel occlusion within 6 hours of presentation, and may be considered in selected patients up to 16 to 24 hours of last known normal. Atrial fibrillation is a frequent cause of stroke and medical management should entail anticoagulation for patients with a CHAD2 score of >2. Aspirin is not considered effective for stroke prevention in patients with atrial fibrillation.

How well did you know this?
1
Not at all
2
3
4
5
Perfectly
21
Q

For each condition, select the most likely associated finding (A-E).
Tolosa-Hunt syndrome
Answers:
A. Trigeminal nerve palsy
B. Facial nerve palsy
C. Pupillary dilation
D. Optic nerve atrophy
E. Ocular motor nerve palsy

A

Ocular motor nerve palsy

According to the International Headache Society (IHS) 3rd guidelines Tolosa-Hunt syndrome is a clinical combination of unilateral periorbital pain with cranial nerve 3, 4 or 6 palsy. Cranial nerve palsy occurs within 2 weeks of headache onset. The causes of this sydrome classically include aneurysm, trauma, tumour, or inflammation affecting the cavernous sinus however the IHS guidelines specifically refer to a granulomatous cause. Optic, trigeminal, facial or parasymapthetic nerves can also be involved but are not required to meet the diagnostic criteria.

How well did you know this?
1
Not at all
2
3
4
5
Perfectly
22
Q

A 35-year-old woman comes to the emergency department because of new onset left hemiparesis.
Past medical history is significant for three miscarriages and a deep venous thrombosis ten years
ago. CT scan of the head and MR imaging of the brain show new areas of ischemic stroke. Her
workup should include which of the following tests?
Answers:
A. heparin-PF4 antibodies
B. TSH
C. von-willebrand factor
D. anticardiolipin antibodies
E. factor X deficiency

A

anticardiolipin antibodies

Antiphospholipid syndrome is an autoimmune, hypercoagulable state caused by antiphospholipid antibodies that can be primary or secondary to other autoimmune diseases such as Systemic Lupus Erythematosus. Diagnostic criteria include a clinical event related to thrombosis or pregnancy complications and two positive blood test results for lupus anticoagulant, antiapolipoprotein antibodies, or anti-cardiolipin antibodies. Treatment consists of anticoagulation therapy to prevent further thrombotic events.

How well did you know this?
1
Not at all
2
3
4
5
Perfectly
23
Q

The cranial nerve most commonly involved in CNS sarcoidosis is the
Answers:
A. CN III
B. CN IV
C. CN V
D. CN VI
E. CN VII

A

CN VII

The facial nerve is most commonly involved with CNS sarcoidosis and is seen in 25-50% of cranial neuropathy cases associated with CNS sarcoidosis. Cranial neuropathy of neurosarcoidosis can involve one or multiple cranial nerves simultaneously. CN III, IV, V, and VI neuropathies are rarely seen with CNS sarcoidosis.

How well did you know this?
1
Not at all
2
3
4
5
Perfectly
24
Q

A 24-year-old woman is referred for evaluation because of recurrent seizures refractory to medical therapy. MR images are shown. Which of the following is the most likely nonhemorrhagic postoperative complication following microsurgical resection of this lesion?
Answers:
A. Wound infection
B. Permanent mutism
C. Temporary akinesia
D. Deep vein thrombosis
E. Right leg weakness

A

Temporary akinesia

The correct answer is temporary akinesia. Resection of this lesion, which is located in the left superior frontal gyrus, is often associated with supplementary motor area syndrome, the clinical sequelae of which include global akinesia on the contralateral side, with preserved muscle strength and mutism. This syndrome typically completely resolve within weeks to months – although the timeline is variable, a full recovery is typically observed within about 3 months. The incidence of SMA syndrome after dorsomedial prefrontal lobe surgery ranges widely in the literature but has been reported at anywhere from 25% to near 100%. The higher the proportion of the anatomical SMA that is resected, the higher chance of observing the clinical syndrome. Permanent mutism is not a correct answer. Mutism is associated with SMA syndrome, but the condition is temporary and spontaneous speech tends to recover suddenly, but patients may have difficulties with speech fluency problems for a longer period of time. A full recovery can be expected in most cases. Wound infection is not the correct answer. The expected incidence of infection after craniotomy for brain tumors is 2-4%. Deep vein thrombosis is not the correct answer. The incidence of DVT after craniotomy for brain tumor in national database analyses is approximately 2.5%. Right leg weakness could be related to a right ACA infarction and is not the correct answer. The incidence of symptomatic ischemic stroke has been reported to occur in approximately 15% of patients undergoing resection of low grade gliomas, most commonly among patients undergoing recurrent surgery or with insular lesions.

How well did you know this?
1
Not at all
2
3
4
5
Perfectly
25
Q

A right-handed 55-year-old man has agraphia, alexia, acalculia, finger agnosia, right-left
disorientation, and difficulty spelling. Which of the following areas of the brain is the most likely site
of a lesion?
Answers:
A. Insula
B. Left pars triangularis
C. Calcarine Sulcus
D. Left Angular Gyrus
E. Parahippocampal Gyrus

A

Left Angular Gyrus

The constellation of agraphia, alexia, acalculia, finger agnosia, and right-left disorientation represents pure Gerstmann Syndrome, and typically occurs as a result from malfunction of cortex in the region of the dominant angular gyrus. The syndrome rarely appears in isolation. The insulae are involved in consciousness and are linked to emotion and regulation of body homeostasis. The triangular part of the dominant inferior frontal gyrus is a component of Broca’s area and is responsible for speech production. The peripheral visual field is located along the anterior portion of the calcarine sulcus and the central visual field is located in the posterior portion. The parahippocampal gyrus lies within the medial temporal lobe surrounding the hippocampus. It is important role in both spatial memory and navigation.

How well did you know this?
1
Not at all
2
3
4
5
Perfectly
26
Q

Which of the following findings is suggestive of a lateral medullary syndrome?
Answers:
A. decreased pain and temperature sensation from ipsilateral body
B. ipsilateral oculomotor palsy
C. contralateral parkinsonian rigidity
D. hoarseness
E. contralateral hemiparesis

A

hoarseness

Hoarseness is a common finding among patients with lateral medullary syndrome (Wallenberg’s syndrome) reflecting injury of the nucleus ambiguus. Additional bulbar symptoms include dysphonia, dysphagia, dysarthria, and decreased gag reflex. Loss of temperature and pain sensation typically occcurs ipsilaterally in the face and contralaterally in the body. Ipsilateral occulomotor palsy, contralateral parkinsonian rigidity, and contralateral hemiparesis are seen in Weber syndrome, a midbrain stroke syndrome involving the cerebral peduncle and ipsilateral fascicles of the oculomotor nerve.

How well did you know this?
1
Not at all
2
3
4
5
Perfectly
27
Q

An 80-year-old man with a history of dementia and prior intracerebral hemorrhage is found
unresponsive. A CT scan of the head is shown. Which of the following is the histopathologic
hallmark of this patient’s most likely condition?
Answers:
A. a-synuclein deposition
B. neurofibrillary tangles
C. Chronic lymphocytic inflamation
D. perivascular pseudorossettes
E. b-amyloid deposition

A

b-amyloid deposition

Cerebral amyloid angiopathy is a frequent cause of intracerebral hemorrhage and its characterized by its lobar distribution in elderly patients. Cerebral amyloid angiopathy is characterized by perivascular deposition of b-amyloid in small and middle sized vessels. a-synoclein is a protein commonly found in Parkinson’s disease. Neurofibrillary tangles are found in Alzheimer’s disease. Perivascular pseudorosettes are characteristic of ependymomas. Chronic lymphocytic inflamation is seen in inflammatory etiologies such as chronic lymphocytic inflammation with pontine perivascular enhancement responsive to steroids (CLIPPERS).

How well did you know this?
1
Not at all
2
3
4
5
Perfectly
28
Q

Which of the following is a feature of the syndrome of alexia without agraphia?
Answers:
A. ideomotor apraxia
B. abulia
C. hemiplegia
D. right homonymous hemianopia
E. left homonymous hemianopia

A

right homonymous hemianopia

Alexia without agraphia is commonly caused by a left PCA infarct. It is often associated with right homonymous hemianopia. Ideomotor apraxia is characterized by the inability to correctly imitate hand gestures and voluntarily mime tool use. It is seen with injuries to the dominant parietal and premotor areas. Abulia is characterized by a lack of will or initiative and can be seen as a disorder of diminished motivation. It can be seen in frontal lobe injuries. Hemiplegia describes paralysis on one side of the body and can be seen in a range of injuries spanning the brain and spinal cord.

How well did you know this?
1
Not at all
2
3
4
5
Perfectly
29
Q

A 55-year-old woman with a history of type 2 diabetes mellitus is evaluated for a two-day history of
diplopia and ptosis of the left eye. Physical examination shows left ptosis; the left eye is deviated
down and out, and the pupil is 5 mm in diameter and unreactive. She is awake and alert, and her
neurological examination is otherwise nonfocal. Which of the following is the most likely diagnosis?
Answers:
A. Left posterior communicating artery aneurysm
B. Giant cavernous ICA aneurysm
C. Tolosa-Hunt syndrome
D. Diabetic third nerve palsy
E. Uncal herniation

A

Left posterior communicating artery aneurysm

A left posterior communicating artery aneurysm is the most likely of the listed causes for the patient’s symptoms, and is the correct answer. Any compressive lesion of the third nerve will cause similar symptoms. Direct compression of the nerve from the outside causes a third nerve palsy that also involves the pupil (i.e. the pupil is dilated and unreactive). The third cranial nerve has two parts: outer parasympathetic fibers that supply the ciliary muscles and sphincter; and inner somatic fibers that supply levator palpebrae superioris and extraocular muscles. The extraocular muscles supplied by the third nerve include the superior rectus, middle rectus, inferior rectus, and inferior oblique. Diabetic third nerve palsy causes ischemia of CNIII and is the most common cause of an acquired unilateral CNIII palsy. However, it is not the correct answer. Diabetic CNIII palsy is typically pupil sparing (i.e., the pupil will remain small) since the diabetic lesion is ischemic, affecting the vasa nervorum and thus sparing the pupillary fibers. Tolosa-Hunt syndrome is a rare painful ophthalmoplegia due to cavernous sinus inflammation. It is usually a clinical diagnosis of exclusion, and pain is a key feature. It typically responds to steroid treatment. Uncal herniation will cause a third nerve palsy that is similar to that caused by a PCOM aneurysm, however, this is typically caused by a compressive lesion such as an extradural hematoma, which would be associated with decreased LOC and other symptoms. A giant cavernous ICA aneurysm is not the correct answer. Although these may present with cranial neuropathies, it is more likely that multiple cranial neuropathies will be observed rather than an isolated CNIII palsy. Furthermore, the third nerve palsy will classically not produce a dilated pupil because the sympathetics that dilate the pupil are paralyzed along with the nerve itself. A sixth nerve palsy is typically the first cranial neuropathy associated with a giant ICA aneurysm.

How well did you know this?
1
Not at all
2
3
4
5
Perfectly
30
Q

Which of the following common cardiac pathologies is most often associated with cerebral
embolism?
Answers:
A. Atrial fibrillation
B. Congestive heart failure
C. Recent myocardial infarction
D. Mechanical prosthetic valve
E. Mitral rheumatic stenosis

A

Atrial fibrillation

Atrial fibrillation most frequently results in cerebral embolism followed by myocardial infarction (1.5% - 25%). Myocardial infarction (2.5%) and congestive heart failure (2%) can also result in thromboembolism, however much less commonly than atrial fibrillation. Likewise mitral rheumatic stenosis and prosthetics heart valve are other rare causes of cerebral embolism.

How well did you know this?
1
Not at all
2
3
4
5
Perfectly
31
Q

Which of the following findings is most likely in a patient with a hemorrhage from a cranial dural arteriovenous fistula?
Answers:
A. Venous drainage into spinal perimedullary veins
B. Cortical venous drainage with more than one draining vein
C. Cortical venous drainage with venous aneurysm
D. Venous drainage into a dural venous sinus with retrograde flow
E. Supply to the fistula from the intracranial, as well as the extracranial, circulation

A

Cortical venous drainage with venous aneurysm

The correct answer is cortical venous drainage with venous aneurysm. Venous aneurysm, ectasia, stricture or a pseudophlebitic pattern are all evidence of venous hypertension and predispose to hemorrhagic presentation. Cortical venous drainage, alone more so than in combination with flow into a sinus, is a high-risk feature. Another angiographic feature that predisposes dAVFs to rupture is sinus thrombosis, which can be antegrade (usually thrombosis of the draining sigmoid sinus) and/or retrograde (e.g. superior sagittal sinus thrombosis). Patients who present with aggressive symptoms including progressive neurological signs are at higher risk of developing hemorrhage. Venous drainage into a dural venous sinus with retrograde flow represent an intermediate risk dAVF. Fistulae that drain into a sinus are generally lower risk than those that have cortical venous drainage, even if there is retrograde flow in the sinus. Venous drainage into spinal perimedullary veins represents a Cognard Type 5 dAVF. This is most often associated with progressive symptoms of myelopathy related to venous congestion/venous hypertension in the medulla or cervical cord. Although it is considered a high-risk feature and would be associated with an elevated risk of hemorrhage compared with a low grade dAVF, these lesions are rare. Cortical venous drainage with more than one draining vein is rare but can occasionally be seen at surgery or at endovascular intervention. In this case, the multiple routes of venous drainage would reduce, rather than increase, the risk of hemorrhage. Supply to the fistula from the intracranial, as well as the extracranial, circulation is not in and of itself a risk factor for hemorrhage. Commonly, transverse-sigmoid sinus fistulae can parasitize arterial supply from the artery of Bernasconi & Cassinari, a branch of the meningohypophyseal trunk of the ICA. This dural branch is not usually amenable to endovascular embolization and may necessitate a transvenous approach for complete cure in these cases.

How well did you know this?
1
Not at all
2
3
4
5
Perfectly
32
Q

Which of the following complications is most likely to occur as a result of a difficult surgical clipping
of an unruptured right choroidal artery aneurysm?
Answers:
A. Aphasia
B. Hemianesthesia
C. Contralateral hemianopia
D. Hemiparesis
E. Hemineglect

A

Hemiparesis

Hemiparesis is the most common neurological deficit that can happen after inadvertently clipping of the anterior choroidal artery during surgery for anterior choroidal artery aneurysms. This is due to the infarct of the posterior limb of the internal capsule. Hemianesthesia can also happen however this often this often recovers in majority of the case after time. Infarction is less frequent in the thalamus, midbrain, temporal lobe, and lateral geniculate body territories of the AChA. Homonymous upper-quadrant anopia, hemianopia, or upper- and lower-quadrant sector anopsia can be present. A homonymous defect in the upper and lower visual fields sparing the horizontal meridian indicates a lesion in the lateral geniculate body in the territory of the AChA. Aphasia does not typically happen from AChA infarct.

How well did you know this?
1
Not at all
2
3
4
5
Perfectly
33
Q

A patient has the new onset of expressive aphasia six days after clipping of a ruptured middle cerebral artery aneurysm. Which of the following is the most likely cause?
Answers:
A. Subdural hematoma
B. Cerebral vasospasm
C. Thromboembolism
D. Seizure
E. Aneurysm rebleeding

A

Cerebral vasospasm

Six days after SAH it is unlikely for the patient to suffer from other complications as compared to vasospasm. Risk of vasospasm peaks at day 7-10. Subdural hematoma is less likely to happen in such a delayed fashion. Aneurysm rebleeding is unlikely to cause expressive aphasia without significant headache and nausea and other symptoms. Seizure is a possibility however unlikely to result in isolated expressive aphasia. Thromboembolic cause is also much less likely this far out from surgery.

How well did you know this?
1
Not at all
2
3
4
5
Perfectly
34
Q

In a patient with an intracranial dural arteriovenous fistula, which of the following features is most
predictive of increased risk of hemorrhage?
Answers:
A. Transverse-sigmoid sinus junction location
B. Aggressive symptoms
C. Direct cortical venous drainage
D. Age > 50 years
E. Associated aneurysms

A

Direct cortical venous drainage

Direct cortical venous drainage is significantly associated with an increased risk of hemorrhage from the dAVF. Aggressive symptoms at presentation are also associated with an increased risk of hemorrhage of dAVF, however this risk is less than that associated with direct cortical venous drainage. Previous hemorrhage from the dAVF poses the highest risk for subsequent rehemorrhage. Frontal dAVF and the fistulas associated with sagittal sinus are associated with increased risk of hemorrhage as compared to other locations. Carotid-cavernous fistulas have a low risk of hemorrhage due to many venous outflow channels. Age more than > 50 years is inconsistently associated with increased risk of hemorrhage. Generally dAVF are not associated with aneurysms.

How well did you know this?
1
Not at all
2
3
4
5
Perfectly
35
Q

A 65-year-old patient is brought to the emergency department because of a 45-minute history of
acute onset gaze deviation, aphasia, and right-sided hemiplegia. Non-contrast CT scan is negative
for intracranial hemorrhage or other acute processes. Which of the following is the most
appropriate next step in management?
Answers:
A. intravenous thrombolysis
B. intraarterial thrombolysis
C. endovascular intervention
D. medical management
E. intravenous thrombolysis followed by endovascular intervention

A

intravenous thrombolysis followed by endovascular intervention

According to the American Heart Association (AHA) guidelines on management of acute ischemic stroke, intravenous thrombolysis is recommended in elegible patients that present within 4.5 hours of ischemic stroke symptom onset. Endovascular intervention with mechanical thrombectomy should be performed in patients with large vessel occlusion within 6 hours of symptom onset, and may be considered in selected patients with acute ischemic stroke within 16 to 24 hours of last known normal.

How well did you know this?
1
Not at all
2
3
4
5
Perfectly
36
Q

Which of the following structures is indicated by the arrow in the photograph of a right pterional
craniotomy shown?
Answers:
A. Membrane of Liliequist
B. Lamina terminalis
C. Optic chiasm
D. Cerebellar tentorium
E. Opticocarotid cistern

A

Lamina terminalis

The lamina terminalis is a thin sheet of gray matter and pia mater stretches upward between the optic chiasm and rostrum of the corpus callosum. It forms the anterior wall of the third ventricle. Its opening allows entry into the third ventricle and release of significant amount of CSF. This facilitates aneurysm dissection without excessive brain retraction. The membrane of Liliequist is an important landmark during dissection of the interpeduncular cisterns. It is stretched between the mammillary bodies and the dorsum sellae and laterally bordered by the oculomotor nerves. The cerebellar tentorium or tentorium is an extension of the dura mater that separates the cerebellum from the inferior portion of the occipital lobes. The opticocarotid cistern is accessed between the optic nerve and the internal carotid artery.

How well did you know this?
1
Not at all
2
3
4
5
Perfectly
37
Q

Occlusion of which of the following arteries is the cause of the stroke in the MR image shown?
Answers:
A. anterior choroidal artery
B. recurrent artery of Heubner
C. subcallosal artery
D. pericallosal artery
E. posterior choroidal artery

A

subcallosal artery

The MR image shows infarction of the left anterior column of the fornix. Typical presentation consists of memory deficit including amnesia with impairment of delayed recall. The subcallosal artery is a perforating branch of the anterior communicating artery and is an important vascular supply to the anterior column of the fornix, while the posterior choroidal branches of the posterior cerebral artery are an important vascular supply to the body of the fornix.

How well did you know this?
1
Not at all
2
3
4
5
Perfectly
38
Q

Projection axons in the olfactory tract originate in which of the following cells?
Answers:
A. Olfactory tract
B. Olfactory cells
C. Mitral cells of olfactory bulb
D. Piriform cortex
E. Entorhinal cortex

A

Mitral cells of olfactory bulb

Bipolar primary olfactory neurons originate in the olfactory cells within the olfactory epithelium, and project to secord order neurons in the olfactory bulb called mitral cells. Axons from mitral cells leave the olfactory bulb through the olfactory tract located under the frontal lobe to the primary olfactory cortex (piriform cortex), which is located on the inferior surface of the temporal lobe. The entorhinal cortex is an area of the brain’s allocortex, located in the medial temporal lobe, whose functions include being a widespread network hub for memory, navigation, and the perception of time.

How well did you know this?
1
Not at all
2
3
4
5
Perfectly
39
Q

Which of the following structures is most responsible for processing proteins for cellular secretion?
Answers:
A. Lysosomes
B. Nucleolus
C. Smooth endoplasmic reticulum
D. Golgi apparatus
E. Rough endoplasmic reticulum

A

Golgi apparatus

The Golgi apparatus is responsible for packing proteins produced by the rough endoplasmic reticulum into vesicles that are then released at the cell membrane. Lysosomes are membranebound organelles that contain hydrolytic enzymes that can break down many kinds of biomolecules. The nucleolus is the site of ribosome biogenesis. The endoplasmic reticulum (ER) is made up of two subunits – rough endoplasmic reticulum (RER), and smooth endoplasmic reticulum (SER). The two types of ER share many of the same proteins and engage in activities such as synthesis of certain lipids and cholesterol.

How well did you know this?
1
Not at all
2
3
4
5
Perfectly
40
Q

In the endoscopic view of the floor of the third ventricle shown, which of the following structures is
indicated by the arrow?
Answers:
A. Mammillary bodies
B. Infundibular recess
C. Lamina terminalis
D. Thalamus
E. Pineal gland

A

Infundibular recess

The infundibular recess is pictured. The endoscope is advanced through the foramen to visualize the third ventricle. The floor proper extends from the optic chiasm to the inlet to the cerebral aqueduct. The lamina terminalis lies anteriorly and rostrally. Immediately below the lamina terminalis, the optic chiasm will be apparent as a horizontal white or yellow band. Inferior to the chiasm is the infundibular recess, a brown or red dimple in the ependyma which gives rise to the pituitary stalk inferiorly. The infundibular recess is almost always visualized, is always midline, and serves as an anatomic landmark. The mammillary bodies lie posterior to the infundibular recess and are part of the posterior part of the hypothalamus. The third ventrile lies between the medial aspects of both thalami. The pineal gland lies posterior to the third ventricle.

How well did you know this?
1
Not at all
2
3
4
5
Perfectly
41
Q

A 36-year-old woman postpartum day 10 presents with a one-week history of headache and mild
confusion. MR imaging shows patchy diffusion restriction with surrounding edema and
intraparenchymal hemorrhage in the same region. MR venogram shows left transverse sinus
thrombosis. Which of the following is the most appropriate initial treatment for this patient?
Answers:
A. Warfarin
B. Aspirin
C. Heparin
D. Mechanical thrombectomy
E. Alteplase

A

Heparin

Both the American Stroke Association and European Stroke Association guidelines recommend commencing anticoagulation with heparin, even in the setting of venous sinus thrombosis causing intracranial hemorrhage. Thrombolysis and thrombectomy are reserved for cases of deteriorating neurological condition and commenced on a case by case basis. Warfarin is not suitable for initial anticoagulation due to the time taken to reach therapeutic levels. There is debate on whether Low Molecular Weight Heparin or Unfractionated Heparin are more appropriate, and the evidence for either is weak. The European guidelines favor low Molecular Weight Heparin due to lower rates of hemorrhagic complications, however the American Stroke Association states that either is appropriate.

How well did you know this?
1
Not at all
2
3
4
5
Perfectly
42
Q

Before intravenous administration of recombinant tissue plasminogen activator to treat a patient
with acute ischemic stroke, which of the following studies is most appropriate to obtain?
Answers:
A. PET
B. MR DWI
C. CT perfusion
D. CT angiogram
E. Non contrast CT

A

Non contrast CT

American Stroke Association guidelines on Acute Ischaemic Stroke 2018 recommend non-contrast CT imaging to rule out Intracranial Hemorrhage before commencing IV alteplase. Imaging to demonstrate large vessel occlusion or ischaemic reversibility are not required before commencing thrombolysis.

How well did you know this?
1
Not at all
2
3
4
5
Perfectly
43
Q

A 20-year-old man is evaluated because of a one-week history of intensely painful and recurrent
headaches accompanied by nasal congestion. The headaches develop in the right periorbital
region and last approximately 80 minutes. Which of the following findings is most likely to be found
on examination of the eyes during or soon after the headaches?
Answers:
A. Extra-ocular muscle palsy
B. Ptosis
C. Mydriasis
D. Miosis
E. Conjunctival injection

A

Conjunctival injection

Cluster headaches are a type of trigeminal autonomic cephalgia.They are orbital, supraorbital or temporal and last up to 180 minutes. They typically occur in men aged 20-40 years. They are associated with conjunctival injection, tearing, nasal congestion, facial/eyelid swelling, miosis or ptosis. Conjunctival injection and tearing are the most common features found in up to 80% of cases.

How well did you know this?
1
Not at all
2
3
4
5
Perfectly
44
Q

Which of the following venous drainage patterns of a dural arteriovenous malformation is most
predictive of an increased risk of hemorrhage?
Answers:
A. Drainage into a cortical vein with venous ectasia
B. Drainage into spinal perimedullary veins
C. Drainage into a dural sinus with retrograde flow in the sinus
D. Drainage into a dural sinus with retrograde flow in the sinus and cortical venous reflux
E. Drainage into a cortical vein

A

Drainage into a cortical vein with venous ectasia

This question asks you to know the Cognard classification of dural arteriovenous fistulae. There are various classification schemes for dAVFs, most commonly the Borden and the Cognard classifications. In each of the classification schemes, higher grades are associated with a higher rate of hemorrhagic presentation.
The correct answer to the question is drainage into a cortical vein with venous ectasia. This represents a Cognard grade 4 (Borden 3) lesion and was associated with the highest risk for hemorrhagic presentation in Cognard’s original description, and has been supported by more modern series as well. Drainage into a dural sinus with retrograde flow in the sinus (Borden 1, Cognard 2a) is a relatively low risk lesion; although there is retrograde flow in the sinus (implying occlusion of the ipsilateral sigmoid sinus), there is no evidence of venous strain or venous hypertension in this case and the hemorrhage risk is not particularly elevated. Drainage into a cortical vein (Borden 3, Cognard 3) does have a higher risk of hemorrhage than fistulae with drainage into a sinus, but the risk is lower compared with cases in which there is also venous ectasia.

Drainage into a dural sinus with retrograde flow in the sinus and cortical venous reflux (Borden 2, Cognard 2a+b) represents an intermediate risk lesion and is not the correct answer. Drainage into spinal perimedullary veins (Borden 3, Cognard 5) is not the correct answer. This is considered a high-risk feature, but these rare lesions most commonly present with evidence of myelopathy.

How well did you know this?
1
Not at all
2
3
4
5
Perfectly
45
Q

During assessment of stroke risk in a patient with atrial fibrillation and the subsequent need for
anticoagulation, in addition to a history of hypertension and diabetes mellitus, which of the
following should be considered?
Answers:
A. hypotension
B. Gender
C. neuropathy
D. congestive heart failure
E. pulmonary hypertension

A

congestive heart failure

The CHADS2 score is a scoring system used to assess the thromboembolic risk in patients with atrial fibrillation based on data from several clinical trials using five common stroke risk factors: congestive heart failure (1 point), hypertension (1 point), age>75 years (1 point), diabetes (1 point), and prior stroke (2 points). Patients with a CHADS2 score ≥ 2 are considered at high risk and may benefit from medical management with oral anticoagulation.

46
Q

A 62-year-old right-handed woman presents with intact speech, but she is unable to read after a
presumed embolic stroke. She is able to write words but cannot read them immediately after
writing them. However, she is able to recognize whole words that are spelled out for her when
spoken by another person. Her infarction is likely in which of the following arterial territories?
Answers:
A. Thalamogeniculate perforators
B. Right middle cerebral artery
C. Left middle cerebral artery
D. Right posterior cerebral artery
E. Left posterior cerebral artery

A

Left posterior cerebral artery

This patient has a clear history of alexia without agraphia aka pure alexia. The lesion is located in the left visual cortex and the splenium of corpus callosum which disconnects the right visual cortex from the left angular gyrus. The vessel supplying this territory will be the left posterior cerebral artery.

47
Q

Which of the following is the most likely predictor of clinically significant vasospasm in a patient
who has sustained a subarachnoid hemorrhage?
Answers:
A. Smoking
B. Hypertension
C. Thick cisternal clot
D. Hunt Hess Grade
E. Young age

A

Hunt Hess Grade

Hunt Hess grade, thick cisternal clot, smoking, hypertension and young age are all the risk factors for vasospasm. Hunt and Hess grade 4 and 5 patients, however have the highest risk of developing vasospasm.

48
Q

In a patient with Spetzler-Martin grade V arteriovenous malformation (AVM), which of the following
radiographic findings is most predictive of an increased risk of hemorrhage?
Answers:
A. Nidal aneurysms
B. Exclusive deep venous drainage
C. Deep location
D. Female sex
E. Older patient age

A

Exclusive deep venous drainage

Although all of the above answer choices including exclusive deep venous drainage, deep location, female sex, older patients and nidal aneurysms have been reported to be associated with increased risk of hemorrhage from an AVM, the most consistent feature reported is the exclusive deep venous drainage. Previous hemorrhage from a ruptured AVM poses the highest risk for subsequent re-rupture for the 1st year after hemorrhage, then the risk comes down to the prerupture risk. Annual hemorrhage rate for AVMs with deep venous drainage ranges from 2.4% to 5.4%. The increased pressure of the deep venous system and resultant increased pressure gradients across the AVM nidus have long been postulated to affect hemorrhage rates for these lesions.

49
Q

Which of the following language deficits is most often encountered acutely after damage to the
dominant supplementary motor area?
Answers:
A. Impaired repetition
B. Impaired spontaneous speech
C. Loss of naming
D. Akinetic mutism
E. Facial agnosia

A

Impaired spontaneous speech

Supplementary Motor area aphasia, occuring after unilatleral damage to the dominant hemisphere, generatlly manifests as reduced spontaneous speech, with intact repetition, naming, and comprehension. Akinetic mutism generally occurs only with bilateral SMA damage. Facial agnosia (prosopagnosia) is characterized by the inability to recognize faces and is seen with injury to the fusiform gyrus.

50
Q

During histologic examination of brain tissue at autopsy, a gray matter area at the level of the facial
colliculus composed of medium-size neurons containing melanin granules is identified. Which of
the following is the most likely source of these cells?
Answers:
A. Locus Coeruleus
B. Red Nucleus
C. Abducens nerve nucleus
D. Motor nucleus of the V CN
E. Raphe Nuclei

A

Locus Coeruleus

CNS neurons containing norepinephrine are found only in the pons and medulla. Most are located in the locus ceruleus (Latin for “blue spot”), a collection of pigmented cells located near the floor of the fourth ventricle. The pigmented neurons contain neuromelanin, accounting for the blueblack appearance of the locus ceruleus in unstained brain tissue. The Locus coerules synthesizes Noradrenaline and is located in the posterior area of the rostral pons, in the lateral floor of the fourth ventricle. The red nucleus is a structure in the rostral midbrain involved in motor coordination. The red nucleus is pale pink due to the presence of iron. The trigeminal motor nucleus contains motor neurons that innervate muscles of the first branchial arch, namely the muscles of mastication, the tensor tympani, tensor veli palatini, mylohyoid, and anterior belly of the digastric. This nucleus is located in the mid-pons. The raphe nuclei are a moderate-size cluster of nuclei found in the brain stem. They have 5-HT1 receptors and decrease the release of serotonin. The abducens nucleus is the originating nucleus from which the abducens nerve (VI) emerges. This nucleus is located beneath the fourth ventricle in the caudal portion of the pons, medial to the sulcus limitans. The abducens nucleus along with the internal genu of the facial nerve make up the facial colliculus, a hump at the caudal end of the medial eminence on the dorsal aspect of the pons.

51
Q

The mass of choroid plexus that protrudes through the lateral aperture of the fourth ventricle
(foramen of Luschka) lies
Answers:
A. on the anterior surface of the glossopharyngeal and vagus nerves
B. on the posterior surface of the glossopharyngeal and vagus nerves
C. on the lateral surface of the trigeminal nerve
D. on the lateral surface of the hypoglossal nerve
E. on the medial surface of the hypoglossal nerve

A

on the posterior surface of the glossopharyngeal and vagus nerves

The lateral recess of the fourth ventricle communicates with cerebellomedullary cistern through the foramen of Luschka. The choroid plexus that projects from the foramen of Luschka sits on the posterior surface of the glossopharyngeal and vagus nerves. The rootlets forming the hypoglossal nerves arise in the posterior wall of the premedullary cistern between the medullary pyramids and the inferior olives. The trigeminal nerve arises from the midpons and courses through the superolateral portion of the cerebellopontine cistern

52
Q

Repetition is most likely to be normal in a patient with which of the following aphasic syndromes?
Answers:
A. Expressive
B. Receptive
C. Transcortical
D. Conduction
E. Word Salad

A

Transcortical

Transcortical aphasias occur when either Wernicke’s or Broca’s areas are isolated from the brain through lesioning, resulting in intact repetition, but impaired comprehension or expression, respectively. The other aphasia syndromes listed demonstrate impaired repetition. Word salad is confused or unintelligible mixture of seemingly random words and phrases. It is a sign seen with Wernicke’s (receptive) aphasia as well as with psychiatric conditions such as schizophrenia.

53
Q

Brain-stem infarction associated with ipsilateral deafness is usually caused by occlusion of the
Answers:
A. Superior Cerebellar Artery
B. Posterior Inferior Cerebellar Artery
C. Thalamoperforators
D. Anterior Inferior Cerebellar Artery
E. Paramedian basilar perforators

A

Anterior Inferior Cerebellar Artery

The dorsal and ventral cochlear nuclei in the brainstem are located laterally at the junction of the pons and the inferior cerebellar peduncle. Hearing loss forms part of the lateral pontine syndrome (Marie-Foix syndrome) due to occlusion of the Anterior Inferior Cerebellar Artery. This syndrome also includes facial palsy, hemiplegia, ataxia, and pain/temperature loss. Lateral medullary syndrome (Wallenberg syndrome) is secondary to occlusion of the Posterior Inferior Cerebellar Artery and is comprised of vertigo, nystagmus, ipsilateral Horner’s, contralateral pain/temperature loss, as well as dysphonia/dysphagia/dysarthria.

54
Q

An 81-year-old man sustains a left hemispheric transient ischemic attack and is found to have 70%
stenosis of the supraclinoid carotid artery. Which of the following is the most appropriate
management?
Answers:
A. Aspirin
B. Dual antiplatelet with aspirin and clopidrogrel
C. Warfarin
D. Angioplasty
E. Angioplasty and Stenting

A

Dual antiplatelet with aspirin and clopidrogrel

In the CLAIR study (The Clopidogrel Plus Aspirin Versus Aspirin Alone for Reducing Embolization in Patients With Acute Symptomatic Cerebral or Carotid Artery Stenosis), patients on dual antiplatelet therapy had fewer TIA events. Based on SAMMPRIS, the current American Heart Association guidelines for secondary stroke prevention specify that for patients with recent stroke or transient ischemic attack (within 30 days) that is attributed to severe stenosis (70%–99%) of a major intracranial artery, dual anti- platelet therapy for 90 days might be reasonable. Dual antiplatelet therapy after 3 months should be followed with a single agent. Endovascular interventions including angioplasty and angioplasty and stenting are reserved for patients having 2 or more strokes. Aspirin alone is inferior to dual antiplatelet therapy.

55
Q

Which of the following factors contraindicates the use of intravenous thrombolysis with tissue
plasminogen activator (tPA) for management of patients with acute ischemic stroke?
Answers:
A. head trauma 6 months prior
B. platelet count <100,000/mm3
C. Systolic blood pressure of 165mmHg
D. cranial surgery 6 months prior
E. glucose levels below 80 mg/dL

A

platelet count <100,000/mm3

According to the American Heart Association (AHA) guidelines, absolute contraindications for intravenous thrombolysis with rtPA are the following: history of ICH, severe uncontrolled hypertension (systolic>185 mm Hg or diastolic>110 mm Hg), < 3 month history of serious head trauma, spinal/cranial surgery or stroke, thrombocytopenia (platelet count <100 000/mm3), presence of an active bleeding diathesis or coagulopathy, intra-axial intracranial neoplasm, aortic arch dissection, GI malignancy or bleed within 21 days, full treatment dose of low molecular weight heparin within the previous 24 hours, direct thrombin inhibitors or direct factor Xa inhibitors, and glucose levels below 50 mg/dL.

56
Q

A patient develops the sudden onset of hemiplegia with contralateral facial paralysis and inward
deviation of the eye. Which of the following arteries is most likely to be occluded?
Answers:
A. Anterior Inferior Cerebellar Artery
B. Paramedian basilar perforators
C. Posterior Inferior Cerebellar Artery
D. Superior Cerebellar Artery
E. Thalamoperforators

A

Paramedian basilar perforators

Hemiplegia, contralateral facial palsy and abducens palsy describe Foville syndrome (medial pontine syndrome). This is caused by occluded basillar perforators. Anterior Inferior Cerebellar Artery occlusion causes Marie-Foix syndrome (lateral pontine syndrome) which includes facial palsy and hemiplegia but also ataxia, pain/temperature loss and hearing loss. The other answers describe medullary or midbrain syndromes.

57
Q

Brown-Séquard syndrome is characterized by which of the following findings on physical
examination?
Answers:
A. contralateral loss of prioprioception below the level of the lesion
B. contralateral loss of light touch below the level of the lesion
C. paralysis below the level of the lesion
D. bilateral loss of pain and temperature sensation
E. contralateral loss of pain and temperature sensation below the level of the lesion

A

contralateral loss of pain and temperature sensation below the level of the lesion

Brown-Séquard syndrome results in contralateral loss of pain and temperature sensation below the level of the lesion due to damage to the ascending lateral spinothalamic tract which crosses 2 to 3 levels above the level of a given respective dorsal root. Also seen is ipsilateral loss of proprioception, touch, and vibration sense below the lesion due to damage to the ascending dorsal columns, ipsilateral upper motor neuron spastic paralysis below the lesion due to damage to the descending lateral corticospinal tracts, and ipsilateral loss of motor and sensory function at the level of the injured segments due to direct damage to ventral and dorsal grey matter.

58
Q

A decrease in which of the following senses is most characteristic of superficial siderosis?
Answers:
A. Olfactory sensations
B. Sensorineural hearing
C. Temperature changes
D. Pain sensations
E. Vibration sense

A

Sensorineural hearing

The significant vulnerability of the eighth cranial nerve to siderosis may be explained by its long glial segment and the longer length of hemosiderin deposition affecting function. The other cranial nerve that is often affected by siderosis is cranial nerve 1 causing abnormal smell sensations, but not as often as the eight cranial nerve. The pigmentation from superficial siderosis has a predilection for the superior vermis, crests of the cerebellar folia, basal frontal lobe, temporal cortex, brainstem, spinal cord, nerve roots, and cranial nerves I and VIII. As the name indicates it affects the superficial layers of the brain and spinal cord which are in contact with CSF. Thus often these patients develop hearing loss and cerebellar ataxia. Other sensations including temperature, pain and vibration are typically not affected. Siderosis of the central nervous system is thought to be caused by bleeding into the subarachnoid space, which may be due to trauma, tumor, arteriovenous malformations, or an unknown cause.

59
Q

Lateral medullary syndrome secondary to a posterior inferior cerebellar artery infarction is most
likely to cause which of the following?
Answers:
A. Lingual palsy
B. Sensory loss
C. Ataxia
D. Horners
E. Vertigo

A

Sensory loss

Lateral medullary syndrome (Wallenberg syndrome) is comprised of vertigo with nystagmus (due to inferior vestibular nucleus and pathways), ipsilateral Horner’s (due to sympathetic fibers), contralateral pain/temperature loss (due to spinothalamic tract), and dysphonia/dysphagia /dysarthria (due to different nuclei and fibers of the IX and X nerves). Sensory loss is the most common feature (90%) which can be a combination of hemibody and/or trigeminal distributions. The other symptoms are all found in 70-75% of cases.

60
Q

Which of the following terms most accurately describes the loss of the red reflex on funduscopic
examination after acute subarachnoid hemorrhage?
Answers:
A. Corneal scarring
B. Vitreous hemorrhage
C. Retinal detachment
D. Ciliary body hemorrhage
E. Orbital infarction

A

Vitreous hemorrhage

Vitreous hemorrhage is the correct answer. The eponym Terson syndrome is commonly used to refer to any intraocular hemorrhage that is seen after subarachnoid hemorrhage (SAH), intracerebral hemorrhage or traumatic brain injury. This can be vitreous hemorrhage, or bleeding in the sub-hyaloid, subretinal space, or beneath the internal limiting membrane. The most common location of bleeding after SAH, however, is vitreous hemorrhage. The finding may be seen in up to 20% of cases of SAH, 10% of ICH and 3% of TBI. The mechanism is due to sudden increase in
ICP, which is transmitted along the optic nerve sheath, compressing the central retinal vein and causing rupture of thin retinal capillaries. Fundoscopic examination is the gold standard for diagnosis, and loss of red reflex is seen in 20% of eyes with vitreous hemorrhage. Ciliary body hemorrhage causes anterior chamber hemorrhage, which causes uncomplicated hyphema that resolves in 5-6 days. This is not related to the loss of red reflex in SAH. Papilledema is a sign of increased intracranial pressure, resulting in swelling of the optic disc and optic nerve head edema. This could be seen in SAH but does not result in loss of the red reflex. Retinal detachment can be a consequence of vitreous hemorrhage but is not the most accurate description for the syndrome of vitreous hemorrhage which is the primary finding explaining the loss of the red reflex. Corneal scarring does cause loss of red reflex, but it is not particularly associated with subarachnoid hemorrhage.

61
Q

In the endoscopic view of the right foramen of Monro shown, which of the following structures is
indicated by the arrow?
Answers:
A. Thalamus
B. Choroid plexus
C. Fornix
D. Septum pellucidum
E. Lamina terminalis

A

Fornix

The foramen of Monro lies in the venous angle between the septal vein (medial) and the thalamostriate vein (lateral). Both veins join the internal cerebral vein. Between these veins lies the choroid plexus which runs rostral to the thalamus. The column of the fornix forms the anterior wall of the foramen, and the anterior pole of the thalamus forms the posterior wall. The septum pellucidum is a thin membrane separating the anterior horns of the left and right lateral ventricles. The lamina terminalis lies anteriorly and rostrally within the third ventricle.

62
Q

Horner syndrome is most likely caused by damage to which of the following?
Answers:
A. Ciliary ganglion
B. Superior ganglion of vagus nerve
C. Otic ganglion
D. Superior rectus muscle
E. Stellate ganglion

A

Stellate ganglion

The stellate ganglion is formed by the cervicothoracic sympathetic fibers, and approximately lies at C7. Injury to this ganglion interrupt sympathetic innervation of the orbit, and can result in Horner’s syndrome. The ciliary ganglion serves as the site of synapse for the parasympathetic nerves innervating the eye. Adie’s tonic pupil is characterized by an idiopathic, sudden-onset denervation of the ciliary body and iris sphincter, followed by a slow abnormal reinnervation. This results in a sudden fixed, dilated pupil and loss of accommodation in that eye. The superior ganglion of the vagus nerve (jugular ganglion) is a sensory ganglion located within the jugular foramen. Pain in the external auditory canal (otalgia) can in rare cases be due microvascular conflict between the posterior inferior cerebellar artery and the auricular branch of the vagus nerve travelling through the superior ganglion. The otic ganglion is a small parasympathetic ganglion associated with the glossopharyngeal nerve located below the foramen ovale in the infratemporal fossa. It innervates the parotid gland for salivation. The superior rectus muscle is an extraoccular muscle in the orbit that elevates the eye and is innervated by the superior division of the oculomotor nerve.

63
Q

Which of the following occurs with medial medullary syndrome but not with lateral medullary
syndrome?
Answers:
A. Nystagmus
B. Ataxia
C. Bulbar dysfunction
D. Hypoglossal palsy
E. Horner syndrome

A

Hypoglossal palsy

Medial medullary syndrome (Dejerine syndrome) is comprised of hypoglossal palsy and contralateral hemisensory and hemiplegia. This is due to loss of the medial lemniscus, corticospinal tracts and hypoglossal nucleus. Lateral medullary syndrome (Wallenberg syndrome) is comprised of vertigo with nystagmus (due to inferior vestibular nucleus and pathways), ipsilateral Horner’s (due to sympathetic fibers), contralateral pain/temperature loss (due to spinothalamic tract), and dysphonia/dysphagia/dysarthria (due to different nuclei and fibers of the IX and X nerves).

64
Q

Following cardiac catheterization for evaluation of heart valve disease, a 67-year-old man reports
loss of sensation of his left face, arm, and leg. Other than the sensory loss, his findings on
neurological examination are normal. Which of the following structures is most likely to show
evidence of a stroke on imaging in this patient?
Answers:
A. postcentral gyrus
B. thalamus
C. posterior limb of the internal capsule
D. precentral gyrus
E. anterior limb of the internal capsule

A

thalamus

Pure sensory stroke (PSS) is a well-defined clinical entity characterized by prominent hemisensory manifestations without other major neurologic signs. Lacunar infarct in the ventroposterior nucleus of the thalamus is the most common cause of pure sensory stroke. Although rare, cases of PSS have also been reported with strokes involving the brainstem, internal capsule, cerebral cortex.

65
Q

A 75-year-old man is evaluated because of a two-month history of headaches and tenderness to
palpation over the right temporal artery. Which of the following is the most likely cause of this
patient’s headaches?
Answers:
A. skull neoplasm
B. tension headaches
C. migraine headaches
D. temporal arteritis
E. cluster headaches

A

temporal arteritis

Temporal arteritis (or giant cell arteritis) is a systemic inflamatory vasculitis of midium-to-large sized vessels characterized by neurologic and ophtalmologic manifestations leading to potentially irreversible vision loss if left untreated. It is considered the most common vasculitis affecting adult patients in western countries and often affect adults >50 years old. Diagnosis is cofirmed by histopathology showing inflammatory infiltrate consisting of CD4+ T-cells and macrophages. Constitutional symptoms are the most common presenting symptoms followed by headaches, jaw claudication, tenderness or enlargement of the temporal artery, and visual symptoms from ischemic optic neuropathy secondary to involvement of the ophthalmic artery. Treatment consists mainly of oral corticosteroids.

66
Q

Which of the following neurological deficits is most commonly caused by inadvertent clip occlusion of the anterior choroidal artery at its origin?
Answers:
A. Hemianopsia
B. Contralateral hemiparesis
C. Hemianesthesia
D. Tremor
E. Aphasia

A

Contralateral hemiparesis

The correct answer is contralateral hemiparesis. The anterior choroidal artery is usually the first branch after the posterior communicating artery and consists of a parenchymal, cisternal segment and a plexal segment that passes through the choroidal fissure to supply the choroid plexus. The cisternal segment supplies the optic tract, cerebral peduncle, uncus, lateral geniculate body, anterior perforated substance (globus pallidus and posterior limb of the internal capsule), anterior portion of the temporal lobe, hippocampus, dentate gyrus, fornix and pulvinar. It is the posterior limb of the internal capsule that is responsible for the contralateral hemiparesis, and since the posterior limb of the internal capsule is only poorly collateralized (by lenticulostriate branches of the MCA), contralateral hemiparesis is the most common consequence of inadvertent AChA occlusion during microsurgical clipping. The arm is typically more severely affected than the leg, although both arm, face and leg may be equally affected. Aphasia may be part of the anterior choroidal artery syndrome but is less consistently observed than hemiparesis. In addition, it is primarily seen only with occlusion of the left AChA. Hemianesthesia and hemianopsia are part of the anterior choroidal artery occlusion syndrome first described by Foix in 1925. However, modern series of complications after clipping of anterior choroidal artery aneurysms identify these deficits in a minority of patients, and inconsistently between patients. Tremor is not a correct answer. In fact, deliberate occlusion of the anterior choroidal artery was advocated by some surgeons in the 1950s as a treatment for advanced Parkinson disease as the hemiparesis associated with AChA infarction appeared to have a beneficial effect on the Parkinsonian tremor. This has not, of course, lasted into the modern neurosurgical era.

67
Q

Paresthesias in all four limbs, ataxia, decreased ankle jerks, and bilateral extensor plantar
responses are most characteristic of which of the following disorders?
Answers:
A. Friedreich’s ataxia
B. Subacute combined degeneration
C. Vitamin E deficiency
D. Guillain-Barre
E. Schilder’s Disease

A

Subacute combined degeneration

Subacute combined degeneration typically presents as a slowly progressive spastic paresis, and can sometimes include positive Babinski signs, and results from deficiency of vitamin B12. Friedreich ataxia is an inherited neurologic condition that progressively impairs muscle coordination resulting in ataxia. Gradual loss of strength and sensation in the arms and legs, spasticity, and impaired speech are also seen. Vitamin E deficiency is rare. Patients may present with symptoms of ataxia, difficulty with upward gaze, and hyporeflexia. Guillain-Barre is an autoimmune condition affecting the peripheral nervous system typically arising a 1-4 weeks after respiratory or gastrointestinal infection. It classically presents with an acute ascending and often rapidly progressive symmetrical weakness, areflexia, and relatively mild sensory abnormalities. Schilder’s disease is a rare progressive demyelinating disorder usually beginning in childhood. The disorder is a variant of multiple sclerosis and symptoms include dementia, aphasia, seizures, personality changes, poor attention, tremors, balance instability, incontinence, muscle weakness, headache, vomiting, and vision and speech impairment.

68
Q

Constructional apraxia found on patient examination most likely indicates damage to which of the
following cortical areas?
Answers:
A. Left parietal lobe
B. Right parietal lobe
C. Left frontal lobe
D. Right frontal lobe
E. Left temporal lobe

A

Right parietal lobe

Constructional apraxia is the inability to copy visually presented information and is most often seen in patients with right parietal lesions. This is likely secondary to disruption of higher-order visuospatial processing,

69
Q

The risk for rerupture of a cerebral aneurysm is greatest at which of the following periods after
initial rupture?
Answers:
A. Post-bleed days 4-12
B. 0-24 hours
C. 24-48 hours
D. 48-72 hours
E. Within the first 2 weeks

A

0-24 hours

The risk of rerupture after aneurysmal subarachnoid hemorrhage is highest in the first 24 hours after the index rupture. This was first established by Kassell et al in the period before early aneurysm surgery and led to a revolution in management of patients with SAH in the 1980s and 1990s which emphasized a shift towards early definitive aneurysm treatment (within the first 24 hours) (International Cooperative Study on the Timing of Aneurysm Surgery). Mortality from aneurysm rebleeding is more severe than initial treatment, and has been reported at up to 60%. In modern practice, in which early aneurysm treatment is routine, the timeline of rebleeding risk is less well established but follows a similar pattern. In a recent series of 574 patients with SAH, the daily risk of rebleeding among patients who had not already experienced rebleeding was 2.6% on day 0, 2.1% on day 1, 0.3% on day 2, 0.7% on day 3, 0.1% on day 4 and a similar low rate thereafter during the initial hospital stay. The risk of aneurysm rebleeding in the period of 24-48 hours is less than in the initial 24 hours after rupture. The risk of aneurysm rebleeding in the period of 48-72 hours is less than in the initial 24 hours after rupture. The risk of aneurysm rebleeding within the first two weeks, when averaged over that entire period, is also less than in the initial 24 hours after rupture. Post-bleed days 4-12 represent the peak risk for vasospasm, which is not associated with a higher risk of aneurysm rebleeding, but can make endovascular or surgical treatment of an untreated aneurysm higher risk for peri-procedural complications.

70
Q

In a left hemisphere-dominant individual, a right parietal lesion would result in which of the
following?
Answers:
A. Acalculia
B. agraphia
C. dressing apraxia
D. tactile agnosia
E. right-left confusion

A

dressing apraxia

Non-dominant (usually right) parietal lobe lesions can cause dressing apraxia, anosognosia, and topographic memory loss. Tactile agnosia is the inability to identify an object by active touch of the hands without other sensory input. It is seen with parietal lobe lesions of either the right or left hemisphere. Tactile agnosia, agraphia (inability to write), right-left confusion, and acalculia (inability to perform simple arithmetic problems) are the classic signs seen with Gerstmann syndrome reflecting dominant inferior parietal lobe injury.

71
Q

Which of the following best describes the sensory loss in a patient with an infarct caused by
occlusion of the posterior inferior cerebellar artery?
Answers:
A. loss of proprioception
B. loss of pain and temperature sensation in ipsilateral face and ipsilateral hemibody
C. loss of pain and temperature sensation in contralateral face and contralateral hemibody
D. loss of pain and temperature sensation in ipsilateral face and contralateral hemibody
E. loss of pain and temperature sensation in contralateral face and ipsilateral hemibody

A

loss of pain and temperature sensation in ipsilateral face and contralateral hemibody

The posterior inferior cerebellar artery (PICA) supplies the lateral medulla oblongata and occlusion may cause a lateral medullary stroke or Wallenberg Syndrome. Sensory deficit in Wallenberg syndrome is mainly due to involvement of the descending trigeminal tract causing impaired ipsilateral pain and temperature sensation in the face, as well as involvement of the spinothalamic tract causing imparied pain and temperature sensation in the contralateral hemibody.

72
Q

Complete sectioning of the corpus callosum most commonly produces which of the following
effects on language ability during neurological examination?
Answers:
A. Finger agnosia
B. Inability to recognize faces
C. Confabulation
D. Inability to repeat phrases
E. Inability to name objects

A

Inability to name objects

Disconnection syndrome, also known as callosal or split brain syndrome, consists of the inability to name objects although one is capable of recognizing them. Prosopagnosia is the inability to recognize familiar faces, including one’s own face. It is associated with injury to the fusiform gyrus and may be seen with PCA infarcts and hemorrhages of the inferomedial temporo-occipital region. Patient with conduction aphasia are unable to repeat phrases. When speech comprehension and speech production remain intact, it is thought to arise from specific damage to the arcuate fasciculus. Patients with confabulation produce fabricated, distorted, or misinterpreted memories about themselves or the world. Despite contradictory evidence, the patients are generally very confident about their recollections. It is classically seen in Wernicke–Korsakoff syndrome, a common manifestation of thiamine deficiency caused by alcohol use disorder. Finger agnosia is the inability to distinguish, name, or recognize the fingers. It can be seen as part of Gerstmann syndrome in the setting of dominant angular gyrus and posterior parietal lesions.

73
Q

A 55-year-old man with a history of hypertension, hyperlipidemia, and smoking presents with
recurrent spells of headache and ataxia when lifting heavy objects. He recently lost consciousness
after throwing a baseball with his left arm. Occlusion of which of the following arteries is the most
likely finding on angiography?
Answers:
A. Basilar
B. Atlantic vertebral
C. Distal subclavian artery
D. Proximal subclavian artery
E. Foraminal vertebral artery

A

Proximal subclavian artery

The patient has numerous vascular risk factors and posterior circulation symptoms which occur during exertion, notably upper limb exertion. The most likely diagnosis is subclavian steal syndrome which occurs when there is stenosis of the subclavian artery before the branching of the vertebral artery. The vertebral artery is the first branch of the subclavian artery. Poor blood flow to the upper limb is compensated for with retrograde flow down the vertebral artery into the subclavian. When increased blood flow to the upper limb occurs during exertion a greater proportion of the cerebral blood is diverted down the vertebral artery and the patient experiences symptoms of vertebro-basilar insufficiency. Bowhunters syndrome also causes vertebro-basillar
insufficiency but is related to head rotation which is not present in this history.

74
Q

Paresthesias in all four limbs, ataxia, decreased ankle jerks, and bilateral extensor plantar
responses are most characteristic of which of the following disorders?
Answers:
A. Friedreich’s ataxia
B. Subacute combined degeneration
C. Vitamin E deficiency
D. Guillain-Barre
E. Schilder’s Disease

A

Subacute combined degeneration

Subacute combined degeneration typically presents as a slowly progressive spastic paresis, and can sometimes include positive Babinski signs, and results from deficiency of vitamin B12. Friedreich ataxia is an inherited neurologic condition that progressively impairs muscle coordination resulting in ataxia. Gradual loss of strength and sensation in the arms and legs, spasticity, and impaired speech are also seen. Vitamin E deficiency is rare. Patients may present with symptoms of ataxia, difficulty with upward gaze, and hyporeflexia. Guillain-Barre is an autoimmune condition affecting the peripheral nervous system typically arising a 1-4 weeks after respiratory or gastrointestinal infection. It classically presents with an acute ascending and often rapidly progressive symmetrical weakness, areflexia, and relatively mild sensory abnormalities. Schilder’s disease is a rare progressive demyelinating disorder usually beginning in childhood. The disorder is a variant of multiple sclerosis and symptoms include dementia, aphasia, seizures, personality changes, poor attention, tremors, balance instability, incontinence, muscle weakness, headache, vomiting, and vision and speech impairment.

75
Q

In a newborn who has hydrocephalus and a myelomeningocele, which of the following is the most
likely location of the CSF obstruction?
Answers:
A. Posterior fossa
B. Foramen of Monro
C. Temporal horns
D. Aqueduct of Sylvius
E. Arachnoid granulations

A

Posterior fossa

McLone and Knepper developed the unified theory of chiari II malformation. They argue that abnormal neurolation leads to CSF leakage into the amniotic sac, which results in hypotension in the developing neural tube. A small posterior fossa then develops resulting in herniation of the rhomboencephalic structures. Hydrocephalus results secondary to maldevelopment of the CSF pathways in the posterior fossa. Obstruction of the foramen of Monro can be seen in anterior third ventricular pathology such as colloid cysts. The aqueduct of Sylvius is the narrowest part of the CSF pathways. Aqueductal stenosis from causes such as obstructive tumors, congenital narrowing, and gliosis can lead to obstructive hydrocephalus. Arachnoid granulations are small protrusions of the arachnoid mater into the dural venous sinuses of the brain, and allow cerebrospinal fluid to exit the subarachnoid space and enter the blood stream. The temporal horns are a part of the lateral ventricles.

76
Q

Panventricular hemorrhage as seen on CT scans is most commonly associated with hemorrhage
from an aneurysm arising from which of the following?
Answers:
A. Anterior choroidal artery
B. Distal PICA
C. ICA terminus
D. Anterior communicating artery
E. Basilar tip

A

Anterior communicating artery

Anterior communicating artery is the correct answer. IVH occurs in 13-28% of ruptured aneurysms, and ACOM aneurysms are most commonly responsible primarily because they are responsible for a higher proportion of ruptured aneurysms compared with other answer choices. After rupture of ACOM aneurysms, IVH occurs due to rupture of blood through the laminar terminalis into the anterior 3rd or lateral ventricles. The presence of IVH has consistently been shown to be a predictor of poor outcome. Anterior choroidal artery aneurysms cause IVH if they are located distal to the choroidal point. This is relatively rare, and since the vessel at this distal point is small, is more likely to result in lateral and third ventricular IVH than panventricular hemorrhage. Basilar terminus aneurysms can cause IVH due to rupture through the floor of the third ventricle, but this is rare. Distal PICA aneurysms can rupture directly into the 4th ventricle through the foramen of Magendie, but are more likely to result in 4th ventricular IVH than panventricular IVH (though it can occur). Overall, these aneurysms are less common than ACOM aneurysms causing IVH. Carotid terminus aneurysms can cause IVH due to rupture though the floor of the third ventricle, but this is rare.

77
Q

A 78-year-old patient is brought to the emergency department because of gaze deviation, aphasia,
and right-sided hemiplegia that began suddenly four hours and 45 minutes ago. Non-contrast CT
scan is negative for intracranial hemorrhage or other acute processes. Which of the following is the
most appropriate next step in management?
Answers:
A. intravenous thrombolysis
B. intraarterial thrombolysis
C. endovascular intervention
D. medical management
E. intravenous thrombolysis followed by endovascular intervention

A

endovascular intervention

According to the American Heart Association (AHA) guidelines on management of acute ischemic stroke, intravenous thrombolysis is recommended only in elegible patients presenting within 4.5 hours of ischemic stroke symptom onset. Endovascular intervention with mechanical thrombectomy should be performed in patients with large vessel occlusion within 6 hours of symptom onset, and may be considered in selected patients with acute ischemic stroke within 16 to 24 hours of last known normal.

78
Q

Which of the following best describes the affected eye position in a patient with a complete diabetic
third cranial nerve palsy?
Answers:
A. Abducted, depressed, mydriasis.
B. Abducted, depressed, miosis.
C. Abducted, miosis and ptosis
D. Adducted, depressed, mydriasis
E. Abducted, depressed, no pupillary change.

A

Abducted, depressed, no pupillary change.

The oculomotor nerve palsy causes a “down and out” eye position, i.e. abducted and depressed, due to unopposed action of the the lateral rectus (CN6) and superior oblique (CN4). Diabetes is a common cause of a non-compressive, aka medical, cause for oculomotor palsy. Blood supply to the nerve is from the outside inwards thus the centre of the nerve is most susceptible to ischaemia/microvascular damage with the parasympathetic fibres on the periphery of the nerve being spared. Thus the parasympathetic control of the pupils remains intact. This is not the case with surgical oculomotor palsy where external compression also damages the parasympathetic fibres and thus the sympathetic fibres are left unopposed to cause pupillary dilation.

79
Q

For each condition, select the most likely associated finding (A-E):
Aneurysm of the posterior communicating artery
Answers:
A. Oculomotor nerve palsy
B. Seizures
C. Retro-orbital pain
D. Asymptomatic
E. Trigeminal nerve palsy

A

Asymptomatic

Unruptured posterior communicating artery aneurysms can cause retro-orbital pain or a new oculomotor palsy when they enlarge and thus may be indications for further investigation or treatment. However the most common presentation of an aneurysm is an incidental finding on imaging done for another purpose. Seizures may occur in large aneurysms irritating the temporal lobe, and facial pain occurs rarely in large aneurysms compressing the trigeminal nerve.

80
Q

A 40-year-old man has a 2-week history of diplopia. A CT scan, MR image, and angiograms are
shown. In addition to left abducens palsy, which of the following is the most likely finding on
physical examination?
Answers:
A. Horner’s syndrome
B. V1 distribution dysesthesia
C. Pupil-involving third nerve palsy
D. Pupil-sparing third nerve palsy
E. Fourth nerve palsy

A

Pupil-sparing third nerve palsy

The most common symptom of an enlarging cavernous aneurysm is diplopia, due to ophthalmoplegia. The third nerve palsy will classically not produce a dilated pupil because the sympathetics that dilate the pupil are paralyzed along with the nerve itself. Diplopia is found in over 60% of patients with symptomatic cavernous aneurysms. Headache, retro-orbital pain or pain in the V1 distribution is the second most common presenting symptom, seen in about 50-60% of cases in retrospective series. Cavernous aneurysms are associated with a higher risk of growth over time than intracranial aneurysms in other locations. Other causes of pupil-sparing oculomotor palsies include diabetic neuropathy, myasthenia gravis, chronic progressive ophthalmoplegia and vasculopathies (e.g. temporal arteritis). This is due to the fact that ischemic lesions affect the vasa nervorum, leading to infarction of the nerve but sparing the parasympathetic fibers that are located on the outside of the nerve. A CN3 palsy that involves the pupil is more commonly a symptom of lesions that compress the nerve after it exits the sinus, including chordomas, clival meningiomas, PCOM aneurysms or distal basilar aneurysms. A fourth nerve palsy is not as commonly seen as an abducens or oculomotor palsy and is most commonly seen in combination with a third nerve palsy. In the largest series of patients with cavernous aneurysms investigating natural history, fourth nerve palsy was never seen without other ocular motor nerve palsies. Dysesthesias in the distribution of the trigeminal nerve are relatively common findings in patients with cavernous aneurysms and are more commonly seen in the V1 distribution than V2 or V3, but are less common overall than oculomotor palsy. Corneal hypoesthesia is probably the most common trigeminal symptom associated with these lesions. Horner’s syndrome, consisting of pupillary constriction and partial ptosis, is a relatively common symptom in extracranial ICA dissection, but is not as common as isolated head or neck pain. In series of cavernous aneurysms, Horner’s pupil, which is caused by ocular sympathetic paresis without third nerve palsy, was very uncommon. The Horner’s syndrome of carotid dissection or aneurysm is postganglionic, and so anhidrosis is not seen.

81
Q

The cerebral angiogram images demonstrate an anterior cerebral artery arteriovenous
malformation (AVM). Which of the following is the principal risk factor indicating a high risk for
hemorrhage in the illustrated AVM?
Answers:
A. Venous varix
B. Single draining vein
C. Deep AVM location
D. Feeding artery aneurysm
E. Diffuse nidus

A

Venous varix

Venous ectasia is the correct answer. This question asks you to identify high risk features for hemorrhage after examining the figure carefully. Remember, the Spetzler-Martin and LawtonYoung grading scales identify risk factors for surgical complications; they do not predict rupture risk of an unruptured AVM. The risks for rupture among patients with unruptured AVMs include angiographic, patient and demographic factors. Increasing age and prior ruptured status are important patient factors for future hemorrhage. Other factors associated with hemorrhage in retrospective cohorts in multivariate analyses include deep brain location, exclusive deep venous drainage, single draining vein, feeding artery aneurysm, and presence of venous varix or varices. Of these, the feature illustrated in this case, is venous varix. The figure demonstrates a small right medial frontal AVM that is fed primarily by a branch of the right ACA. The lateral view demonstrates that there are two main draining veins. One drains anteriorly into the superior sagittal sinus, and the other (major) draining vein drains into the deep drainage system. You can see the internal cerebral vein outlined in the figure. There is a venous varix just at the outflow point from the AVM and prior to the drainage into the internal cerebral vein. Deep AVM location is related to AVM hemorrhage risk, but the AVM illustrated here would not be considered a deep location. Deep location refers to basal ganglia, internal capsule, thalamus or corpus callosum. Feeding artery aneurysm is related to AVM hemorrhage risk, but the DSA illustrated here does not show any feeding artery aneurysm. Single draining vein is related to AVM hemorrhage risk, but as mentioned above this AVM has both superficial and deep venous drainage. Diffuse nidus is one of the elements of the Lawton-Young grading scale which is a supplementation to the Spetzler-Martin scale and helps predict neurological outcome and morbidity after surgery. It is not related to hemorrhage risk.

82
Q

Which of the following is the most appropriate endovascular approach for the management of
patients with acute ischemic stroke from large vessel occlusion?
Answers:
A. Intra-arterial thrombolysis
B. Mechanical thrombectomy with stent-retriever
C. Mechanical thrombectomy with aspiration only
D. Mechanical thrombectomy with aspiration and stent-retriever
E. Mechanical clot disruption with a microwire

A

Mechanical thrombectomy with stent-retriever

The randomized stroke trials and the metanalysis (HERMES) study were done using stent-retriever for mechanical thrombectomy which is the best evidence for mechanical thrombectomy for acute ischemic stroke. Some of the comparative literature between stent-retriever and aspiration show equivalent outcomes; however the evidence is relatively weak for those comparison studies. Aspiration along with stent- retriever is a very reasonable approach as well with good outcomes, however the evidence supporting its use as the primary modality is weak as well. Intra-arterial thrombolysis can be used for symptoms onset less than 4.5 hours, however this has fallen out of favor after the results with stent- retriever were published. Mechanical clot disruption with a microwire results in higher incidence of distal embolization and overall the outcomes are inferior to stent-retriever.

83
Q

Question:
Which of the following is a common finding following occlusion of the posterior inferior cerebellar
artery?
Answers:
A. facial weakness
B. vertigo
C. contralateral hemiplegia
D. contralateral loss of facial pain and temperature
E. ophthalmoplegia

A

vertigo

The posterior inferior cerebellar artery (PICA) supplies the lateral medulla, inferior cerebellar peduncle, and inferior cerbellar hemisphere. PICA occlusion may cause a lateral medullary stroke or Wallenberg Syndrome. Pateints can be affected to a variable degree. A stroke of the lateral medulla typically affects the inferior vestibular nucleus and pathways, fibers of CN IX and X, sympathetic fibers, inferior cerebellar hemisphere, spinocerebellar fibers, inferior cerebellar peduncle, descending trigeminal tract, nucleus of tractus solitarius, and the spinothalamic tract. Common symptoms include vertigo with nystagmus, hiccups, nausea and vomiting, dysphonia, dysarthria, dysphagia, horner syndrome, ipsilateral ataxia, loss of pain and temperature sensation in the ipsilateral face and contralateral arm and leg, and impaired taste.

84
Q

Which of the following characteristics is most likely in a patient who has lateral medullary
syndrome secondary to a posterior inferior cerebellar artery infarct?
Answers:
A. Lingual palsy
B. Sensory loss
C. Ataxia
D. Horners
E. Vertigo

A

Sensory loss

Lateral medullary syndrome (Wallenberg syndrome) is comprised of vertigo with nystagmus (due to inferior vestibular nucleus and pathways), ipsilateral Horner’s (due to sympathetic fibers), contralateral pain/temperature loss (due to spinothalamic tract), and dysphonia/dysphagia /dysarthria (due to different nuclei and fibers of the IX and X nerves). Sensory loss is the most common feature (90%) which can be a combination of hemibody and/or trigeminal distributions. The other symptoms are all found in 70-75% of cases.

85
Q

A 43-year-old woman has a dense right hemiparesis immediately after undergoing successful clipping of a carotid bifurcation aneurysm complicated only by minor aneurysmal bleeding. Which of the following is the most likely cause of this deficit?
Answers:
A. Subarachnoid hemorrhage
B. Delayed Cerebral Ischemia
C. Postoperative subdural hematoma
D. Inadvertent occlusion of the Internal carotid artery
E. Anterior choroidal artery infarct

A

Anterior choroidal artery infarct

The most likely explanation for this patient’s post-op dense hemiparesis is anterior choroidal artery infarct arising from temporary clipping which was likely used during intra-operative rupture. Infarction could have occured either due to vasospasm or direct injury of the anterior choroidal artery from manipulation during temporary clipping, or from ischemia during prolonged trapping of the ICA bifurcation. ICA occlusion with clip is unlikely as the clipping was successful and would also be associated with cortical signs such as language deficits. It is too early to develop delayed cerebral ischemia after subarachnoid hemorrhage as the deficits occured immediately after surgery. Post op subdural hematoma is a possiblity however it would be associated with other signs such as decreased consciousness, seizures, and/or pupillary changes. Subarachnoid hemorrhage in itself would not cause unilateral weakness.

86
Q

A 35-year-old woman comes to the emergency department with new onset of a complex partial
seizure. CT scan shows a lobar hemorrhage, and cerebral angiography shows a venous angioma.
Which of the following is the most likely cause of the hemorrhage?
Answers:
A. Developmental venous anomaly (DVA)
B. Amyloid angiopathy
C. Hypertension
D. Arteriovenous malformation
E. Cavernous malformation

A

Cavernous malformation

The patient presenting with hemorrhage and venous angioma (aka developmental venous anomaly) likely has an associated cavernous malformation. The hemorrhage was likely from the cavernous malformation. Cavernous malformations are angiographically occult. The risk of hemorrhage from DVA is much less likely. Amyloid angiopathy and hypertension are not typically associated with DVA. Arteriovenous malformation would be demonstrated on angiogram with early venous drainage.

87
Q

A 27-year-old woman has a long history of headaches. MR imaging of the brain shows a 2-cm left
posterior frontal lobe arteriovenous malformation with superficial and deep venous drainage. A
multicenter randomized trial that compared observation to intervention demonstrated which of the
following?
Answers:
A. The risk of developing a focal neurological deficit was not significantly different between
the observation and intervention groups.
B. The risk of death from AVM was lower in the observation group compared with the
intervention group
C. There was a lower than expected rate of primary outcome observed in the observation
group
D. The majority of patients in the interventional group were treated with surgery
E. The risk of death or symptomatic stroke from any cause was lower in the observation
group compared with the intervention group

A

The risk of death or symptomatic stroke from any cause was lower in the observation group
compared with the intervention group

This question requires you to be familiar with the ARUBA trial, which was one of the most significant large trials and highest level of evidence that we have regarding approach to management of unruptured cerebral AVMs. It has changed practice throughout the world, and it is important to understand the nuances of this trial, as well as its limitations, in advising patients about the risks and benefits of the various options. The correct answer is that the risk of symptomatic stroke or death from any cause was significantly lower in the observation group compared with the intervention group. The primary outcome of the ARUBA trial was death or symptomatic stroke from any cause, and was seen in 35 patients (30.7%) in the interventional group, compared with 11 patients (10.1%) in the medical group (RR 0.33, 95% CI 0.18-0.61). The risk of death from AVM was not lower in the observation group than in the intervention group. There were 2 patients in the interventional group who had an AVM-related death, compared with none in the medical management group, but this was not significantly different. The risk of developing a focal neurological deficit was significantly different between the groups, with a lower rate observed in the medical management group. Focal deficits were seen in 1 patient in the medical group, compared with 14 patients in the interventional group (p=0.0008). The supposition that there was a lower than expected event rate in the observation rate is incorrect. In fact, one of the criticisms of the ARUBA trial was that there was a much higher rate of major stroke or death in the intervention group than in other large trials and meta-analyses. The primary outcome of symptomatic stroke or death occurred in 30.7% of patients in the intervention group. A large systematic meta-analysis of prior trials reported rates of severe complications of 7.4% for surgery, 6.6% for embolization and 5.1% for radiosurgery, all of which are significantly lower than the rate observed in the ARUBA trial. Only a small minority of patients in the ARUBA trial were treated with surgery, so this choice is incorrect. Among the 114 patients in the trial randomized to treatment, only 5 patients received surgical resection alone. In contrast, 30 patients were treated with embolization alone, radiotherapy alone in 31, or a multimodal approach in 28. Among those treated with multimodal therapy, more were tread with radiotherapy plus embolization than embolization plus surgery. References:

88
Q

Which of the following is the most appropriate first-line therapy for cerebral venous sinus
thrombosis with intracerebral hemorrhage?
Answers:
A. antiplatelet medication
B. anticoagulation
C. surgical evacuation
D. endovascular treatment
E. observation with tight blood pressure control

A

anticoagulation

According to the American Heart Association (AHA) guidelines on management of cerebral venous sinus thrombosis, the first-line treatment of cerebral venous sinus thrombosis is anticoagulation therapy. Intracranial hemorrhage that occurred as the consequence of cerebral sinus thrombosis is not a contraindication for anticoagulation. Endovascular therapy may be considered in patients with absolute contraindications for anticoagulation therapy or failure of initial therapeutic doses of anticoagulant therapy.

89
Q

A 19-year-old man undergoes anterior discectomy and fusion in the upper cervical spine. On
follow-up evaluation, which of the following physical findings is most suggestive of an
intraoperative injury to the right hypoglossal nerve?
Answers:
A. choking when swallowing
B. dysphagia
C. hoarseness
D. tongue deviation to the left
E. tongue deviation to the right

A

tongue deviation to the right

Ipsilateral tongue deviation can occur after injury to the hypoglossal nerve. Dysphagia, choking while swallowing, and hoarseness can be seen with injury to recurrent laryngeal nerve.

90
Q

A 42-year-old man is brought to the emergency department because of a right carotid artery
dissection that resulted in a hemispheric infarct involving the entire territory of the right middle
cerebral artery. MR image shows right uncal herniation. A life-saving hemicraniectomy is
performed. Which of the following is the best predictor of the outcome of the procedure?
Answers:
A. Volume of infarction
B. Age of the patient
C. Craniectomy size
D. Male sex
E. Time to surgery

A

Age of the patient

Age of the patient has been consistently shown to correlate with the outcomes after surgery. Although well-designed randomized studies may demonstrate improved outcomes with early hemicraniectomy, this has not been proven conclusively in any series. In addition to infarct volume, craniectomy area and male sex has not been shown to influence outcome.

91
Q

Brodmann’s area 4 of the cerebral cortex is distinguished by
Answers:
A. location posterior to central sulcus
B. Few pyramidal cells
C. Prominent granular layer
D. Location in pars triangularis
E. Thickest cortical layer

A

Thickest cortical layer

The primary motor cortex corresponds to Brodmann’s area 4, occupying a tapering strip in the precentral gyrus, mostly in the anterior wall of the central sulcus. Area 4 is agranular, the thickest cortex in the brain, and contains a preponderance of large pyramidal cells, including giant pyramidal cells (Betz cells). Broca’s area is located in pars triangularis and is made up of Brodmann’s area 44 and 45.

92
Q

A 70-year-old man experiences his second documented lobar hemorrhage in four years. Gradientecho MR imaging reveals evidence of hemosiderin in two other locations. The patient is not currently on anticoagulant therapy. Which of the following is the most likely cause of the hemorrhage?
Answers:
A. brain tumor
B. dural arteriovenous fistula
C. amyloid angiopathy
D. uncontrolled hypertension
E. arteriovenous malformation

A

amyloid angiopathy

Cerebral amyloid angiopathy is an important cause of lobar intracerebral hemorrhage in older adults and may present with transient neurological symptoms, leukoencephalopathy, and with incidental microbleeds or hemosiderosis on magnetic resonance imaging. Other causes of spontaneous intracranial hemorrhage include hypertensive vasculopathy which usually occurs in arteries that are particullary susceptible to the effects of hypertension given the decrease in vessel caliber from larger parent vessels. Arteriovenous malformations, dural arteriovenous fistulas, aneurysmal bleed, and brain tumors are typically associated with additional findings on imagining.

93
Q

The cell viewed under electron microscopy in the figure shown is generally found adjacent to which
of the following?
Answers:
A. Neurons
B. Astrocytes
C. Microglia
D. Ventricle wall
E. Oligodendocytes

A

Ventricle wall

An ependymal cell is pictured. Ependymal cells line the ventricles and interface with the periventricular white matters. Tight junctions prevent CSF movement across this layer, but novel evidence has suggested a possible role in CSF production and homeostasis. Neurons are electrically excitable cells that communicates with other cells via specialized connections called synapses. They are the main component of nervous tissue. Astrocytes are characteristic starshaped glial cells in the brain and spinal cord. They perform many functions, including biochemical support of endothelial cells that form the blood–brain barrier, provision of nutrients to the nervous tissue, maintenance of extracellular ion balance, regulation of cerebral blood flow, and a role in the repair and scarring process of the brain and spinal cord following infection and traumatic injuries. Microglia are the macrophage cells of the brain an spinal cord. They act as the first and main form of active immune defense in the central nervous system. Oligodendrocytes create myelin sheath to support and insulate axons in the central nervous system.

94
Q

A 73-year-old woman develops a brain-stem syndrome from loss of a paramedian perforator
caused by progressive enlargement of an aneurysm. CT angiograms three years apart are shown.
Which of the following deficits are most likely to be detected on this patient’s neurological
examination?
Answers:
A. ipsilateral facial numbness
B. contralateral hemianesthesia
C. ipsilateral hemiparesis
D. ipsilateral ophthalmoplegia
E. contralateral loss of facial pain and temperature
Correct Answer:

A

ipsilateral ophthalmoplegia

The aneurysm is causing mass effect on the left cerebral peduncle at the midbrain level. Involvement of perforators at this level may result in Weber syndrome from midbrain stroke. Weber syndrome typically involves the substantia nigra, corticospinal fibers, corticobulbar tract, and oculomotor nerve fibers. Common presenting symptoms include CNIII palsy, contralateral hemibody weakness, contralateral parkinsonism, and lower facial muscle wekaness. The aneurysm size on MRI appears larger than what is filling on the vascular imaging with CTA. Therefore, this likely represents a partially thrombosed aneurysm.

95
Q

Which of the following blood vessels is most likely to be involved in trigeminal neuralgia?
Answers:
A. Petrosal vein
B. Superior cerebellar artery
C. Anterior inferior cerebellar artery
D. Basilar artery
E. Posterior cerebral artery

A

Being in the vicinity of trigeminal nerve and due to pulsalite nature of blood flow, the superior cerebellar artery is most often associated with trigeminal neuralgia. Veins can cause trigeminal neuralgia in approximately 10% cases. The transverse pontine vein is the most common offending vein. The basilar artery typically does not result in trigeminal neuralgia unless there is fusiform aneurysm or dolicoectasia. The anterior inferior cerebellar artery is an uncommon cause of trigeminal neuralgia unless associated with an aneurysm or there is an aberrant loop of the vessel.

96
Q

A 46-year-old man develops left face and arm weakness after clip ligation of an anterior
communicating artery aneurysm from a right-sided approach. Inadvertent ligation of which of the
following vessels led to these postoperative symptoms and signs?
Answers:
A. Medial lenticulostriate artery
B. Lateral lenticulostriate artery
C. Anterior communicating artery
D. Anterior choroidal artery
E. Recurrent artery of Heubner

A

Recurrent artery of Heubner

This is a classic question asking about ischemic complications of anterior communicating artery aneurysm clipping. Recurrent artery of Heubner (medial striate artery) is the correct answer. The recurrent artery of Heubner most commonly originates from the junction of the ACA and the ACOM ~60%), followed by proximal A2 (~25%), and less commonly from the A1 (~15%). It curves backwards, paralleling the A1 (hence its name). It gives vascular supply to the head of the caudate, medial globus pallidus, anterior limb of the internal capsule, anterior hypothalamus, nucleus accumbens, part of the uncinate fasciculus, diagonal band of Broca, and basal nucleus of Meynert. The clinical syndrome of occlusion of the recurrent artery of Heubner (for example, by an errant clip intended for an ACOM aneurysm) leads to weakness of contralateral arm and face. Dysarthria and hemichorea may be variably observed. Bilateral occlusion leads to akinetic mutism. Anterior choroidal artery is not the correct answer. Inadvertent anterior choroidal artery occlusion is most commonly a complication of PCOM aneurysm clipping, and classically leads to a triad of contralateral hemiplegia, contralateral hemianesthesia and contralateral hemianopia. In practice, AChA occlusion most commonly leads to contralateral hemiplegia so it matches the clinical syndrome described in the question; however, as mentioned above the AChA is not particularly at risk during ACOM aneurysm clipping. Medial lenticulostriate artery is not the correct answer. The medial lenticulostriate arteries (usually two) arise most commonly from the A1 segment of the ACA and supply the globus pallidus and medial part of the putamen. Some variability in origin is seen in origin, as these vessels can originate from MCA or even from the recurrent artery of Heubner. Infarction in the territory of the medial lenticulostriates may cause contralateral hemiparesis but are more classically associate with transient memory disorders or psychological agitation. In practice, these small vessels are not exposed during a typical dissection for clipping anterior communicating artery aneurysms. Lateral lenticulostriate artery is not the correct answer. The lateral lenticulostriate arteries (usually 5) arise from the proximal MCA, so they would not be particularly at risk during ACOM aneurysm clipping. Lateral lenticulostriate arteries supply the lentiform nucleaus, part of the caudate, and posterior limb of the internal capsule. Most commonly, lateral lenticulostriate infarction causes contralateral hemiplegia. Anterior communicating artery is not the correct answer. Sometimes it is necessary to occlude the anterior communicating artery during clipping (or coiling!) of ACOM aneurysms, but this is not recommended. Perforating arteries arise from the posterior surfaces of the ACOM, and these branches supply the hypothalamus, optic chiasm, anterior commissure, genu of the internal capsule, and the anterior part of the globus pallidus. Infarction of one or more of these small perforating arteries can cause hypothalamic and frontal symptoms such as memory problems, cognitive/executive dysfunction, and electrolyte disturbances. Sometimes, the columns of the fornix can be supplied by these perforators and bilateral injury to these structures leads to Korsakoff’s syndrome.

97
Q

Neurocritical Care Society guidelines for the management of large hemispheric infarction include
which of the following recommendations with respect to decompressive hemicraniectomy (DHC)?
Answers:
A. DHC should be offered as a therapy to improve survival after large hemispheric infarction
regardless of patient age
B. DHC should be performed for patients with large hemispheric stroke after observing
neurological decline
C. DHC should be offered as a therapy to improve functional outcome in patients over the
age of 60
D. DHC should be offered as a therapy to improve functional outcome in patients under the age of 60
E. DHC should not be offered to right-handed patients with left large hemispheric stroke due
to low likelihood of achieving functional independence

A

DHC should be offered as a therapy to improve survival after large hemispheric infarction
regardless of patient age

Decompressive hemicraniectomy (DHC) should be offered as a therapy to improve survival after large hemispheric infarction regardless of patient age. This is the correct answer. The question tests your knowledge of the evidence for decompressive hemicraniectomy in patients with large hemispheric (malignant) stroke, along with guidelines about this issue. Level 1 evidence for the benefit of early decompressive hemicraniectomy after malignant MCA stroke comes from the DESTINY Trial (Decompressive Surgery for the Treatment of Malignant Infarction of the Middle Cerebral Artery) and the DECIMAL Trial (Decompressive Craniectomy in Malignant Middle Cerebral Artery Infarction). Both of these trials were small, but these along with multiple other randomized studies demonstrated significant reduction in mortality with hemicraniectomy compared with medical management in large hemispheric stroke. Patients in these trials were all <60 years of age. DESTINY II included patients 61 years and older, and again showed a significant reduction in mortality in the hemicraniectomy group, but with a significant increase in patients surviving but with poor and very poor neurological outcome. In a pooled analysis, DHC more than doubled the chance of survival from 29 to 78% (ARR 49%, NNT 2). However, there was no significant improvement in functional outcome. DHC should be not offered as a therapy to improve functional outcome in patients either under or over the age of 60. DHC has been shown to improve mortality, but not functional outcome, so these choices are incorrect. The choice regarding right-handed patients with a left sided stroke may be a reasonable clinical approach, but it is not supported by high level evidence and is not a recommendation of the NCC Society guidelines. In fact, two small studies looking at this issue concluded that decompression of a dominant hemisphere was not associated with worse functional outcomes or QOL. DHC should be performed after observing neurological decline is incorrect. DHC is recommended to be performed within 24-48 hours of symptom onset and prior to any herniation symptoms. Several studies have compared early (<24h) with late (>24h) decompression and these, along with DESTINY and DECIMAL trials, suggest a mortality benefit in early decompression (although some studies suggest an increased rate of disability). In balance, the NCC Society guidelines recommend early decompression.

98
Q

Which of the following is most likely to be a result of dolichoectasia of the vertebral arteries and the
basilar artery?
Answers:
A. Vertigo
B. Trigeminal neuralgia
C. Hemifacial spasm
D. Hearing loss
E. Hydrocephalus

A

Hemifacial spasm

Basilar dolichoectasia is defined as a basilar artery diameter >4.5mm. The enlarged, ectatic vessel causes symptoms of compression (cranial nerves, brainstem, 3rd ventricle) or strokes. Infarction is the most common presentation, however the facial nerve is the nerve most commonly compressed, thus causing hemifacial spasm. 3% of patients will have hydrocephalus due to compression of the 3rd ventricle.

99
Q

A 45-year-old man is brought to the emergency department after being found unresponsive at
home. During transportation to the hospital, his condition deteriorated and he was intubated for
airway protection. Examination in the emergency department shows anisocoric pupils with good
light responses. The left pupil is 5 mm in diameter, and the right pupil is 2 mm. Skew deviation is
noted. A CT scan of the head shows a mild diffuse subarachnoid hemorrhage. Which of the
following is the most likely cause of these findings?
Answers:
A. Pontine infarct due to a basilar perforator aneurysm
B. Horner syndrome due to a ruptured paraophthalmic aneurysm
C. Third nerve palsy due to a ruptured PCOM aneurysm
D. Pharmacologic pupil
E. Physiologic anisocoria

A

Pontine infarct due to a basilar perforator aneurysm

The constellation of symptoms here suggests a pontine infarction, which out of the choices given, is most likely due to rupture of a basilar perforator aneurysm with subsequent thrombosis of the perforator. First off, the patient experienced a neurological decline and was unable to protect his airway, suggesting an impairment of consciousness. The eye exam findings also suggest pontomesencephalic insult. Anisocoria with preserved light reflex may have a number of causes, but in this case suggests a brainstem etiology in which there is impairment of the oculomotor sympathetic fibers without complete infarction of the oculomotor nucleus (which would cause a non-reactive pupil). Skew deviation is an abnormality of ocular motility in which the eyes move in opposite directions as the patient looks upward. This is a sign of brainstem or cerebellar infarction, causing aberrant prenuclear vestibular input into the oculomotor nuclei. Horner syndrome is not seen here. In Horner syndrome, the abnormal pupil is small (miosis), and associated with ipsilateral ptosis and anhidrosis. There is no abnormal reaction to light. Horner syndrome may be associated with a large or giant ICA aneurysm, but these are usually cervical or cavernous in location and would not, therefore, be associated with SAH or with this clinical presentation. A third nerve palsy due to a posterior communicating artery aneurysm most commonly causes a compressive oculomotor palsy (“peripheral” third nerve neuropathy) that involves the pupil and causes a large pupil, ptosis, and oculomotor palsy. A PCOM aneurysm may cause isolated CNIII palsy, but the oculomotor examination here is not consistent with this. Furthermore, with a PCOM rupture, you would expect a larger burden of subarachnoid blood. Pharmacologic pupil is a common cause of anisocoria when encountered in the general population, but is not the correct choice here. Inadvertent administration to the eye of medication that affects the pupil, such as phenylephrine, clonidine, or atropine, could be encountered in the field or in the ED and should be kept in the back of your mind as you examine a patient with SAH. However, in this case the pupil would be tonic, and would not react to light. Physiologic anisocoria occurs in about 20% of the population, but is not the best answer here. The difference in pupil size in physiologic anisocoria is usually less than 0.4mm. A skew deviation, as well as reduced consciousness, would not be seen.

100
Q

Which of the following interventions has been shown to reduce the risk of rebleeding after
aneurysm rupture?
Answers:
A. Induced hypothermia
B. Strict blood pressure control with a goal systolic blood pressure of <120
C. Maintenance of euvolemia
D. Tranexamic acid in patients in whom early obliteration of the aneurysm is possible
E. Control of blood pressure using a titratable antihypertensive agent

A

Control of blood pressure using a titratable antihypertensive agent

This question probes your knowledge of medical measures to prevent rebleeding after aSAH. A prime resource for understanding our literature around managing patients with aneurysmal SAH comes from the AHA/American Stroke Association guidelines for the management of aneurysmal subarachnoid hemorrhage. You should read this guideline and be familiar with the major recommendations. Control of blood pressure using a titratable antihypertensive agent is the correct answer to this question. This is one of the key recommendations in the AHA/ASA Guidelines and is a Level 1 recommendation. Nicardipine, labetalol, sodium nitroprusside, and clevidipine have been studied and are all options depending on patient factors and institutional protocols. Maintenance of euvolemia is not the correct answer. Maintenance of euvolemia and normal circulating blood volume is recommended to prevent DCI and symptomatic vasospasm, but it has not been shown to have an effect on rebleeding rates. Strict blood pressure control with a target systolic blood pressure of <120 is not the correct answer. There is general consensus that avoidance of hypertension is beneficial in reducing rebleeding from a ruptured aneurysm, but strict blood pressure parameters have not been well defined (some retrospective series did not identify SBP<160 as a risk factor for rebleeding in multivariate analysis, while others suggested a correlation). The AHA/ASA guidelines recommend maintaining systolic BP <160. Tranexamic acid in patients in whom early obliteration of the aneurysm is possible is not the correct answer. There is some evidence for antifibrinolytic therapy in reducing rebleeding rate in SAH, but it has been investigated in patients in whom early obliteration of the aneurysm is NOT possible. A short course (<72 hours) of aminocaproic acid or tranexamic acid was shown to reduce rebleeding without increasing risk of DCI in a single-center study, with an increased risk of DVT but not of PE. Induced hypothermia is not the correct answer. Systemic hypothermia has been used as a neuroprotective strategy in head injury, ischemic stroke and circulatory arrest, and was evaluated in a multicenter RCT in patients undergoing craniotomy for clipping of ruptured aneurysms. This trial showed that hypothermia was relatively safe but did not lead to reduced mortality or improved outcomes. It has not been shown to reduce rebleeding risk.

101
Q

A 61-year-old man is evaluated for acute onset of a right homonymous hemianopia. Which of the
following vessels, thought to be the conduit for an artery-to-artery embolus, is indicated by the
arrow in the right internal carotid angiogram shown?
Answers:
A. Persistent proatlantal artery
B. Persistent trigeminal artery
C. Otic artery
D. fetal posterior communicating artery
E. persistent hypoglossal artery

A

Persistent trigeminal artery

A persistent trigeminal artery is the most common type of persistent fetal carotid-vertebrobasilar anastomoses: anatomical variants of arterial communications between the anterior and posterior circulations. The persistent trigeminal artery arises from the proximal cavernous internal carotid artery to join the basilar artery and may lead to posterior circulation embolic stroke originating from the carotid circulation. Other carotid-vertebrobasilar anastomoses include the hypoglossal artery, which originates from the internal carotid artery between C1-3 levels traversing the hypoglossal cannal to join the basilar artery, and the otic artery, which originates from the petrous internal carotid artery within the carotid canal and runs through the internal auditory canal to join the proximal basilar artery. The other anastomosis is the proatlantal artery which originates from the common, internal (Type I), or external (Type II) carotid arteries at the C2-4 levels and joins the vertebral artery at the suboccipital region after traversing the foramen magnum.

102
Q

A 76-year-old woman has symptoms of headache, proximal muscle stiffness, and unilaterally
decreased vision. Laboratory studies show an increased erythrocyte sedimentation rate. Which of
the following is the most likely diagnosis?
Answers:
A. Short-lasting, Unilateral, Neuralgiform headache attacks with Conjunctival injection and
Tearing
B. Cluster headache
C. Temporal arteritis
D. Medication overuse
E. Painful optic neuritis

A

Temporal arteritis

Short-lasting, Unilateral, Neuralgiform headache attacks with Conjunctival injection and Tearing (SUNCT) and cluster headaches are unilateral and associated with ocular tearing and injection but no visual loss. Medication overuse is associated with chronic opioid use of which there is no history in this patient. Painful optic neuritis (retrobulbar neuritis) is pain behind the eye(s) with central visual loss typically seen in Multiple Sclerosis. The correct answer is temporal arteritis (giant cell arteritis) based on the proximal myopathy which is a feature of polymyalgia rheumatica, a condition which is commonly associated with temporal arteritis. These patients often have an elevated erythrocyte sedimentation rate, headache, and unilateral vision loss.

103
Q

Which of the following areas is entered when opening the structure under the tip of the white arrow in the photograph shown?
Answers:
A. ambient cistern
B. third ventricle
C. sylvian fissure
D. interpeduncular cistern
E. interhemispheric fissure

A

third ventricle

The lamina terminalis is a thin sheet of gray matter and pia mater stretches upward between the optic chiasm and rostrum of the corpus callosum. It forms the anterior wall of the third ventricle. Its opening allows entry into the third ventricle and release of significant amount of CSF. This facilitates aneurysm dissection without excessive brain retraction. The ambient cistern is a thin, sheet-like extension of the quadrigeminal cistern that extends laterally around the midbrain. The interpeduncular cistern (or basal cistern) is an unpaired CSF-filled subarachnoid cistern located between the cerebral peduncles. It contains the proximal portion of the third cranial nerve and the basilar apex. The interhemispheric fissure lies within the midline and seperates both cerebral hemispheres. It contains the falx cerebri. The sylvian fissure is the subarachnoid space separating the frontoparietal and temporal lobes.

104
Q

A 59-year-old man is evaluated for a three-hour history of diplopia and pain behind his left eye with
moderate paresis of up, down, and medial gaze. Neurological examination shows a left pupil that
is larger than the right pupil and reacts poorly to light. The remainder of the examination shows no
abnormalities. Which of the following studies is the most appropriate next step in diagnosis?
Answers:
A. White blood count
B. ESR, CRP, and calcitonin levels
C. Angiogram
D. MRI with and without contrast
E. Blood glucose levels

A

MRI with and without contrast

Painful ophthalmoplegia consists of periorbital or hemicranial pain with ipsilateral oculomotor nerve (CN III) palsy and is usually caused by aneurysms of the internal carotid artery, trauma, infection, and malignancy. Contrast enhanced MRI has high sensitivity for detection of structural abnormalities and help identify the underlying etiology. Diabetic CN III palsies, also called ischemic, are the most common etiology in adults and patients typically present with ptosis and diplopia with preserved pupillary function and normal imaging. In contrast, patients with compressive CN III palsy typically have a fixed and dilated pupil due to paralysis of sphincter pupillae. While somatic (voluntary) nerve fibers are bundled deep inside CN III, the autonomic (involuntary) fibers surround the somatic fibers and are located on the outside of the nerve and are relatively less prone to ischemic injuries. Tolosa–Hunt is a rare syndrome that presents as an acute onset of unilateral painful ophthalmoplegia affecting CN III, IV, and/ or VI. It is usually idiopathic and is related to non-specific inflammation in the region of the cavernous sinus and/or superior orbital fissure. The pain and motor dysfunction respond to systemic corticosteroids.

105
Q

A 24-year-old woman on oral contraceptives is evaluated for new-onset headache and a
generalized seizure. A CT scan without contrast (Figure 1) and MR images without contrast
enhancement (Figures 2 and 3) are shown. Which of the following is the most appropriate
management?
Answers:
A. Reverse anticoagulants by administering FFP and/or specific reversal agents
B. Take for urgent cerebral angiogram
C. Initiate heparin infusion
D. Reverse antiplatelet medications and administration of platelets
E. Obtain MRI and MRV

A

Initiate heparin infusion

The correct answer is to initiate heparin infusion. The patient has a hemorrhage related to a transverse and superior sagittal sinus thrombosis, and the most important initial step is to start treatment with anticoagulant medication. Figure 1 demonstrates hyperdensity in the region of the left transverse sinus with a low-density area in the temporal lobe which is a likely area of venous infarct. Figures 2 and 3 demonstrate an area of hemorrhagic conversion. The hint in the question stem to clue you in to the diagnosis is that the patient is currently taking oral contraceptive. OCP use, pregnancy, local infection (e.g. otitis media), trauma, malignancy and sickle cell trait (among many other factors) are all associated with an increased risk of the development of cortical or dural sinus thrombosis. The most common locations for dural sinus thromboses are in the SSS and right transverse sinus. In cases of sinus thrombosis, treatment should be initiated immediately and aggressively, as this has been shown to reduce mortality and improve neurological outcomes. Administration of platelets or reversal of antiplatelet agents is usually among the first steps of management for hypertensive or general ICH. However, in this case, the better answer is to initiate heparin first. MRI is an excellent imaging tool for diagnosis and follow-up of cerebral venous thrombosis, but it would not be the best choice here. MRI can differentiate between clots of different ages, as well as differentiate cerebral edema and non-acute hemorrhage. MRV tends to over-estimate the degree of thrombosis. Catheter angiography is useful in diagnosis, although studies suggest that diagnostic accuracy of MRI and DSA are similar. DSA may falsely suggest an occluded transverse sinus when in fact nonopacified blood filling a sinus can mimic a filling defect. DSA can also be used for treatment, although this is typically reserved for patients who are refractory to, or have progressive neurological decline, despite anticoagulation. Reversal of anticoagulant medications and/or administration of specific agents such as andexanet or prothrombin complex concentrate (Kcentra), would not be recommended. Again, this is a recommended early course of action after hypertensive or general ICH. However, among patients with hemorrhage from sinus thrombosis, initiation of heparin therapy is the first step.

106
Q

A previously healthy 40-year-old woman is evaluated because of a two-day history of severe
headache. Her only medication is an oral contraceptive. CT scan and CT angiography of the brain
are shown. Which of the following is the most appropriate next step in management of this patient?
Answers:
A. Reverse antiplatelet medications and administration of platelets
B. Initiate heparin infusion
C. Obtain MRI and MRV
D. Take for urgent cerebral angiogram
E. Reverse anticoagulants by administering FFP and/or specific reversal agents

A

The correct answer is to initiate heparin infusion. The patient has a hemorrhage related to a transverse and superior sagittal sinus thrombosis, and the most important initial step is to start treatment with anticoagulant medication. The figure shows the filling defect in the right transverse sinus, along with the associated venous hemorrhage. The hint in the question stem to clue you in to the diagnosis is that the patient is currently taking oral contraceptive. OCP use, pregnancy, local infection (e.g. otitis media), trauma, malignancy and sickle cell trait (among many other factors) are all associated with an increased risk of the development of cortical or dural sinus thrombosis. The most common locations for dural sinus thromboses are in the superior sagittal sinus and right transverse sinus. In cases of sinus thrombosis, treatment should be initiated immediately and aggressively, as this has been shown to reduce mortality and improve neurological outcomes. Administration of platelets or reversal of antiplatelet agents is usually among the first steps of management for hypertensive or general ICH. However, in this case, the better answer is to initiate heparin first. MRI is an excellent imaging tool for diagnosis and follow-up of cerebral venous thrombosis, but it would not be the best choice here. MRI can differentiate between clots of different ages, as well as differentiate cerebral edema and non-acute hemorrhage. MRV tends to over-estimate the degree of thrombosis. Catheter angiography is useful in diagnosis, although studies suggest that diagnostic accuracy of MRI and DSA are similar. DSA may falsely suggest an occluded transverse sinus when in fact nonopacified blood filling a sinus can mimic a filling defect. DSA can also be used for treatment, although this is typically reserved for patients who are refractory to, or have progressive neurological decline, despite anticoagulation. Reversal of anticoagulant medications and/or administration of specific agents such as andexanet or prothrombin complex concentrate (Kcentra), would not be recommended. Again, this is a recommended early course of action after hypertensive or general ICH. However, among patients with hemorrhage from sinus thrombosis, initiation of heparin therapy is the first step.

107
Q

Once a cerebral arteriovenous malformation becomes symptomatic, either by hemorrhage or by
seizure, the annual risk for hemorrhage is closest to what percentage?
Answers:
A. 0.02
B. 0.5
C. 0.06
D. 0.2
E. 0.14

A

0.06

The annual risk of hemorrhage from an unruptured AVM is about 2% which goes up to about 6% in the first year after AVM rupture and comes back down to about 2-3% after 1st year. Some literature suggests the rate of re-hemorrhage to be 15%, however the most consistent risk quoted is about 6%.

108
Q

A 37-year-old Hispanic woman with intractable epilepsy undergoes surgery for a right frontal
lesion. Pathology shows irregular vascular channels without intervening neural parenchyma. There
are no large feeding arteries or large feeding veins. Her mother has the same disease. Which of
the following is the pattern of inheritance of this disease?
Answers:
A. X-linked dominant
B. Autosomal recessive
C. Autosomal dominant
D. Mitochondrial
E. X-linked recessive

A

Autosomal dominant

The lesion described here is a cavernous malformation, and the correct answer is autosomal dominant. The other described inheritance patterns are incorrect. Hereditary cavernoma syndromes have an autosomal dominant inheritance pattern with variable expressivity. Parents who are affected by the disorder have a 50% chance of passing it on to their children. There are three main types, all of which cause loss-of-function mutations: CCM1 (KRIT1 gene), CCM2 (malcaverin), and CCM3 (PDCD10 gene). CCM1 is more common among patients with a Hispanic background. Mutations in the cavernoma genes cause activation of Rho-A kinase, causing endothelial barrier dysfunction. There is a higher risk of hemorrhage for cavernomas in patients with CCM3 mutations. Multiple CM are common in patients with familial CM syndromes, with up to 50% of patients harboring multiple lesions. Overall, familial CMs account for approximately 6% of cases of cerebral CM. The precise mechanism by which the CCM mutations leads to microvascular malformation remains to be elucidated, and there is some evidence that immune responses to hemorrhage or cytoskeletal abnormalities may also play a role in the pathogenesis. References:

109
Q

A 61-year-old woman has a 9-mm, posterior, projecting, basilar apex aneu

A

Endovascular coiling of the aneurysm

In the setting of SAH from the aneurysm with a favorable neck-to-dome ratio, coiling of the aneurysm is the safest option. Placing an external ventricular drain would be appropriate if the patient had hydrocephalus. Surgical clipping has higher morbidity for basilar tip aneurysms as compared to clipping. Flow diverter stenting is typically not necessary for basilar tip aneurysm and in the setting of rupture not advised due to the need for dual antiplatelet therapy.

110
Q

A healthy 27-year-old woman is admitted to the hospital after having a subarachnoid hemorrhage.
On admission, she is awake, alert, and oriented. Examination shows right oculomotor nerve palsy.
Motor strength is 5/5 throughout. Which of the following Hunt-Hess grades describes her initial
presentation?
Answers:
A. Grade 1
B. Grade 2
C. Grade 3
D. Grade 4
E. Grade 5

A

Grade 2

Isolated cranial nerve deficit without any other neurological deficit corresponds to Hunt-Hess Grade grade 2. In this grade patients can have associated moderate to severe headaches. Mild headaches or asymptomatic is Hunt-Hess Grade grade 1. Drowsiness, confusion or mild focal neurological deficit corresponds to Hunt-Hess Grade grade 3, Stupor (localization to pain) is HuntHess Grade grade 4 and Coma with posturing is Hunt-Hess Grade grade 5.